Fundamental Midterm review - evolve

Ace your homework & exams now with Quizwiz!

Which of the following statements indicate that the new nursing graduate understands ways to remain involved professionally? (Select all that apply.) A. "I am thinking about joining the health committee at my church." B. "I need to read newspapers, watch news broadcasts, and search the Internet for information related to health." C. "I will join nursing committees at the hospital after I have completed orientation and better understand the issues affecting nursing. D. "Nurses do not have very much voice in legislation in Washington, DC, because of the nursing shortage." E. "I will go back to school as soon as I finish orientation."

A. "I am thinking about joining the health committee at my church." Correct B. "I need to read newspapers, watch news broadcasts, and search the Internet for information related to health." Correct C. "I will join nursing committees at the hospital after I have completed orientation and better understand the issues affecting nursing."Correct

A nurse is teaching a community group about ways to minimize the risk of developing osteoporosis. Which of the following statements reflect understanding of what was taught? (Select all that apply.) A. "I usually go swimming with my family at the YMCA 3 times a week." B. "I need to ask my doctor if I should have a bone mineral density check this year." C. "If I don't drink milk at dinner, I'll eat broccoli or cabbage to get the calcium that I need in my diet." D. "I'll check the label of my multivitamin. If it has calcium, I can save money by not taking another pill." E. "My lactose intolerance should not be a concern when considering my calcium intake."

A. "I usually go swimming with my family at the YMCA 3 times a week." Correct B. "I need to ask my doctor if I should have a bone mineral density check this year." Correct C. "If I don't drink milk at dinner, I'll eat broccoli or cabbage to get the calcium that I need in my diet." Correct

The nurse is teaching a patient how to perform a testicular self-examination. Which statement made by the patient indicates a need for further teaching? A. "I'll recognize abnormal lumps because they are very painful." B. "I'll start performing testicular self-examination monthly after I turn 15." C. "I'll perform the self-examination in front of a mirror." D. "I'll gently roll the testicle between my fingers."

A. "I'll recognize abnormal lumps because they are very painful." Correct The examination should be performed monthly in all men 15 years of age and older. Feel for small, pea-size lumps on the front and side of the testicle. Abnormal lumps are usually painless.

The nurse is teaching a patient how to perform a testicular self-examination. Which statement by the nurse is correct? A. "The testes are normally round and feel smooth and rubbery." B. "The best time to do a testicular self-examination is before your bath or shower." C. "Perform a testicular self-examination weekly to detect signs of testicular cancer." D. "Since you are over 40 years old, you are in the highest risk group for testicular cancer."

A. "The testes are normally round and feel smooth and rubbery." Correct Men ages 18 to 24 are in the group at most risk for testicular cancer. Teaching should include normal anatomy. A testicular examination should be planned monthly during a shower since the soap and water ease movement of the fingertips over the skin.

A 10-year-old girl was playing on a slide at a playground during a summer camp. She fell and broke her arm. The camp notified the parents and took the child to the emergency department according to the camp protocol for injuries. The parents arrive at the emergency department and are stressed and frantic. The 10-year-old is happy in the treatment room, eating a Popsicle and picking out the color of her cast. What is the correct order for the nurse's discussion with the parents? 1. "Can I contact someone to help you?" 2. "Your daughter is happy in the treatment room, eating a Popsicle and picking out the color of her cast." 3. "I'll have the doctor come out and talk to you as soon as possible." 4. "Let me help you two calm down a bit so I can take you to your daughter." A. 2, 4, 3, 1 B. 4, 2, 1, 3 C. 3, 1, 4, 2 D. 2, 3, 4, 1

A. 2, 4, 3, 1 Correct First and most important the parents need to know the immediate status of their daughter. Letting them know the situation will help to relieve their immediate stress. Second, helping the parents calm down and reduce their stress will allow them to see their daughter without increasing the 10-year-old's anxiety. Third, let the parents know you recognize their need to talk to the doctor as soon as possible and you will act as their advocate to get that accomplished. Last, but important, you want to ask if there is anyone you can call to help. There may be children who need to be picked up from camp/day care, and a neighbor or grandparent may be able to assist.

The licensed practice nurse (LPN) provides you with the change-of-shift vital signs on four of your patients. Which patient do you need to assess first? A. 84-year-old man recently admitted with pneumonia, RR 28, SpO2 89% B. 54-year-old woman admitted after surgery for fractured arm, BP 160/86 mm Hg, HR 72 C. 63-year-old man with venous ulcers from diabetes, temperature 37.3°C (99.1°F), HR 84 D. 77-year-old woman with left mastectomy 2 days ago, RR 22, BP 148/62

A. 84-year-old man recently admitted with pneumonia, RR 28, SpO2 89% Correct Oxygen saturation is low, indicating a problem with ventilation or diffusion, which is related to the respiratory rate.

The nurse is teaching a patient with poor arterial circulation about checking blood flow in the legs. Which information should the nurse include? (Select all that apply.) A. A normal pulse on the top of the foot indicates adequate blood flow to the foot. B. To locate the dorsalis pedis pulse, take the fingers and palpate behind the knee C. When there is poor arterial blood flow, the leg is generally warm to the touch. D. Loss of hair on the lower leg indicates a long-term problem with arterial blood flow.

A. A normal pulse on the top of the foot indicates adequate blood flow to the foot. Correct D. Loss of hair on the lower leg indicates a long-term problem with arterial blood flow. Correct A normal dorsalis pedis indicates good arterial blood flow to the lower extremities. Chronic loss of arterial flow results in a lack of hair growth and the appearance of shiny tissue. The dorsalis pedis is located along the top of the foot between the great toe and first toe. When there is poor arterial flow, the skin will be cool.

Which of the following properly applies an ethical principle to justify access to health care? (Select all that apply.) A. Access to health care reflects the commitment of society to principles of beneficence and justice. B. If low income compromises access to care, respect for autonomy is compromised. C. Access to health care is a privilege in the United States, not a right. D. Poor access to affordable health care causes harm that is ethically troubling because nonmaleficence is a basic principle of health care ethics. E. Providers are exempt from fidelity to people with drug addiction because addiction reflects a lack of personal accountability. F. If a new drug is discovered that cures a disease but at great cost per patient, the principle of justice suggests that the drug should be made available to those who can afford it.

A. Access to health care reflects the commitment of society to principles of beneficence and justice. Correct B. If low income compromises access to care, respect for autonomy is compromised. Correct D. Poor access to affordable health care causes harm that is ethically troubling because nonmaleficence is a basic principle of health care ethics. Correct Justice is the ethical principle that justifies the agreement to ensure access to care for all, but it does not necessarily clarify how to resolve issues of limited resources such as money or organs available for transplant. Privilege is not an ethical principle. Nonmaleficence means "first do no harm." A lack of care because of poor access causes harm (i.e., no preventive services, no early detection, no risk reduction) and therefore is ethically troubling. The principal of fidelity implies that we agree to ensure access to care even for people whose beliefs and behaviors may differ from our own, including drug addicts.

A 34-year-old man who is anxious, tearful, and tired from caring for his three young children tells you that he feels depressed and doesn't see how he can go on much longer. Your best response would be which of the following? A. "Are you thinking of suicide?" B. "You've been doing a good job raising your children. You can do it!" C. "Is there someone who can help you?" D. "You have so much to live for."

A. Although this sounds abrupt, the patient usually is relieved that you've broached this issue. For safety reasons it is very important to discuss his suicidal thoughts with the patient.

A nurse plans to provide education to the parents of school-aged children and includes which of the following result of children being less physically active outside of school? A. An increase in obesity B. An increase in heart disease C. Higher computer literacy D. Improved school attendance and grades

A. An increase in obesity

Identify behaviors that foster the development of trust. (Select all that apply.) A. Answer the call light promptly. B. Call the patient by first name unless requested otherwise. C. Do all the care as quickly as possible and leave the room so the patient can rest. D. Answer questions honestly. E. Demonstrate competence when doing treatments.

A. Answer the call light promptly. D. Answer questions honestly. E. Demonstrate competence when doing treatments

The nurse prepares to conduct a general survey on an adult patient. Which assessment is performed first while the nurse initiates the nurse-patient relationship? A. Appearance and behavior B. Measurement of vital signs C. Observing specific body systems D. Conducting a detailed health history

A. Appearance and behavior Correct The nurse inspects appearance and behavior first as part of the nursing assessment. As the patient enters the room, the nurse can observe his or her appearance and behavior, noting any unusual choice of clothing or hygiene or any signs of confusion, anxiety, or happiness.

The nurse prepares to conduct a general survey on an adult patient. Which assessment is performed first while the nurse initiates the nurse-patient relationship? A. Appearance and behavior B. Measurement of vital signs C. Observing specific body systems D. Conducting a detailed health history

A. Appearance and behavior Correct The first part of the general survey is assessment of the appearance and behavior of the patient. As you are initiating the nurse-patient relationship, observe gender and race, age, signs of distress, body type, posture, gait, body movement, hygiene and grooming, dress, affect and mood, speech, and signs of patient abuse.

When using ice massage for pain relief, which of the following are correct? (Select all that apply.) A. Apply ice using firm pressure over skin. B. Apply ice until numbness occurs and remove the ice for 5 to 10 minutes. C. Apply ice until numbness occurs and discontinue application. D. Apply ice for no longer than 10 minutes.

A. Apply ice using firm pressure over skin. Correct B. Apply ice until numbness occurs and remove the ice for 5 to 10 minutes. Correct Cold therapies are particularly effective for pain relief. Ice massage involves applying a frozen cup of ice firmly over the skin. When numbness occurs, remove the ice for usually 5 to 10 minutes.

As a nurse prepares to provide morning care and treatments, it is important to question a patient about a latex allergy before which intervention? (Select all that apply.) A. Applying adhesive tape to anchor a nasogastric tube B. Inserting a rubber Foley catheter into the patient’s bladder C. Providing oral hygiene using a standard toothbrush and toothpaste D. Giving an injection using plastic syringes with rubbercoated plungers E. Applying a transparent wound dressing

A. Applying adhesive tape to anchor a nasogastric tube Correct B. Inserting a rubber Foley catheter into the patient’s bladder Correct D. Giving an injection using plastic syringes with rubbercoated plungers Correct Adhesive tape, rubber foley catheters, and rubber-coated plungers should be avoided for patients with latex allergies since they can trigger an allergic or anaphylactic response.

A patient is receiving 5000 units of heparin subcutaneously every 12 hours while on prolonged bed rest to prevent thrombophlebitis. Because bleeding is a potential side effect of this medication, the nurse should continually assess the patient for the following signs of bleeding: (Select all that apply.) A. Bruising Correct B. Pale yellow urine C. Bleeding gums Correct D. Coffee ground-like vomitus Correct E. Light brown stool

A. Bruising Correct C. Bleeding gums Correct D. Coffee ground-like vomitus Correct

Which action(s) are appropriate for the nurse to implement when a patient experiences orthostatic hypotension? (Select all that apply.) A. Call for assistance. B. Allow patient to sit down. C. Take patient’s blood pressure and pulse. D. Continue to ambulate patient to build endurance. E. If patient begins to faint, allow him to slide against the nurse’s leg to the floor.

A. Call for assistance. B. Allow patient to sit down. C. Take patient’s blood pressure and pulse. E. If patient begins to faint, allow him to slide against the nurse’s leg to the floor.

You are floated to work on a nursing unit where you are given an assignment that is beyond your capability. What is the best nursing action to take first? A. Call the nursing supervisor to discuss the situation B. Discuss the problem with a colleague C. Leave the nursing unit and go home D. Say nothing and begin your work

A. Call the nursing supervisor to discuss the situation Correct Alerting the nursing supervisor as a representative of the hospital administration is the first step in providing notice that a problem may exist related to insufficient staffing. This notice serves to share the burden of knowledge of the staffing inequity issues that may create an unsafe patient situation for the hospital and nursing staff.

You are floated to work on a nursing unit where you are given an assignment that is beyond your capability. Which is the best nursing action to take first? A. Call the nursing supervisor to discuss the situation B. Discuss the problem with a colleague C. Leave the nursing unit and go home D. Say nothing and begin your work

A. Call the nursing supervisor to discuss the situation Correct Alerting the nursing supervisor as a representative of the hospital administration is the first step in providing notice that a problem may exist related to insufficient staffing. This notice serves to share the burden of knowledge of the staffing inequity issues that may create an unsafe patient situation for the hospital and nursing staff.

A health care provider writes the following order for an opioidnaive patient who returned from the operating room following a total hip replacement. "Fentanyl patch 100 mcg, change every 3 days." Based on this order, the nurse takes the following action: A. Calls the health care provider, and questions the order B. Applies the patch the third postoperative day C. Applies the patch as soon as the patient reports pain D. Places the patch as close to the hip dressing as possible

A. Calls the health care provider, and questions the order Correct Fentanyl is 100 times more potent than morphine and not recommended for acute postoperative pain.

The nurse is caring for a patient whose calcium intake must increase because of high risk factors for osteoporosis. The nurse would recommend which of the following menus? A. Cream of broccoli soup with whole wheat crackers and tapioca for dessert B. Hamburger on soft roll with a side salad and an apple for dessert C. Low-fat turkey chili with sour cream and fresh pears for dessert D. Chicken salad on toast with tomato and lettuce and honey bun for dessert

A. Cream of broccoli soup with whole wheat crackers and tapioca for dessert Correct The dairy and broccoli in the soup, the whole grain crackers, plus the tapioca are all great sources of calcium.

A patient has been on bed rest for over 4 days. On assessment, the nurse identifies the following as a sign associated with immobility: A. Decreased peristalsis B. Decreased heart rate C. Increased blood pressure D. Increased urinary output

A. Decreased peristalsis Correct

A patient who is having difficulty managing his diabetes mellitus responds to the news that his hemoglobin A1C, a measure of blood sugar control over the past 90 days, has increased by saying, "The hemoglobin A1C is wrong. My blood sugar levels have been excellent for the last 6 months." The patient is using the defense mechanism: A. Denial. B. Conversion. C. Dissociation. D. Displacement.

A. Denial is avoiding emotional stress by refusing to consciously acknowledge anything that causes intolerable anxiety. This patient's statements reflect denial about poorly controlled blood sugars.

During a patient's routine annual physical, she tells you that she has noted that her heart feels like it's " racing, " usually in the later morning, early afternoon, or just before she goes to bed. Her radial pulse rate is 68 beats/min and regular; her blood pressure is 134/82 mm Hg. What additional information is helpful in evaluating the patient's racing heart? (Select all that apply.) A. Dietary habits B. Medication list C. Exercise regimen D. Age, weight, and height

A. Dietary habits Correct B. Medication list Correct Dietary habits may include caffeine fluids and foods that stimulate heart rate. Medication list may include pharmacological agents that increase or decrease heart rate.

A nurse is caring for an older adult who has had a fractured hip repaired. In the first few postoperative days, which of the following nursing measures will best facilitate the resumption of activities of daily living for this patient? A. Encouraging use of an overhead trapeze for positioning and transfer. B. Frequent family visits C. Assisting the patient to a wheelchair once per day D. Ensuring that there is an order for physical therapy

A. Encouraging use of an overhead trapeze for positioning and transfer. Correct

The ethics of care suggests that ethical dilemmas can best be solved by attention to relationships. How does this differ from other ethical practices? (Select all that apply.) A. Ethics of care pays attention to the environment in which caring occurs. B. Ethics of care pays attention to the stories of the people involved in the ethical issue. C. Ethics of care is used only in nursing practice. D. Ethics of care focuses only on the code of ethics for nurses E. Ethics of care focuses only on understanding relationships.

A. Ethics of care pays attention to the environment in which caring occurs. Correct B. Ethics of care pays attention to the stories of the people involved in the ethical issue. Correct Ethic of care focuses on environmental issues affecting care, the narratives of the patients and health care providers, and understanding relationships.

The patient has a fractured femur that is placed in skeletal traction with a fresh plaster cast applied. The patient experiences decreased sensation and a cold feeling in the toes of the affected leg. The nurse observes that the patient's toes have become pale and cold but forgets to document this because one of the nurse's other patients experienced cardiac arrest at the same time. Two days later the patient in skeletal traction has an elevated temperature, and he is prepared for surgery to amputate the leg below the knee. Which of the following statements regarding a breach of duty apply to this situation? (Select all that apply.) A. Failure to document a change in assessment data B. Failure to provide discharge instructions C. Failure to follow the six rights of medication administration D. Failure to use proper medical equipment ordered for patient monitoring E. Failure to notify a health care provider about a change in the patients condition

A. Failure to document a change in assessment data Correct E. Failure to notify a health care provider about a change in the patient’s condition Correct The failure to document a change in assessment data and the failure to notify a health care provider about a change in patient status reflect a breach of duty to the patient.

When a nurse assesses a patient for pain and offers a plan to manage the pain, which principal is used to encourage the nurse to monitor the patient's response to the pain? A. Fidelity B. Beneficence C. Nonmaleficence D. Respect for autonomy

A. Fidelity

When a nurse assesses a patient for pain and offers a plan to manage the pain, which principal is used to encourage the nurse to monitor the patient's response to the pain? A. Fidelity B. Beneficence C. Nonmaleficence D. Respect for autonomy

A. Fidelity Correct Requiring a return to the patient to evaluate the effectiveness of an intervention exemplifies keeping a promise, a concrete example of fidelity.

When designing a plan for pain management for a postoperative patient, the nurse assesses that the patient's priority is to be as free of pain as possible. The nurse and patient work together to identify a plan to manage the pain. The nurse continually reviews the plan with the patient to ensure that the patient's priority is met. Which principle is used to encourage the nurse to monitor the patient's response to the pain? A. Fidelity B. Beneficence C. Nonmaleficence D. Respect for autonomy

A. Fidelity Correct` Fidelity means keeping promises. Keeping the promise in this case includes not just tending to the clinical need but evaluating the effectiveness of the interventions.

Select statements that apply to the proper use of a cane. (Select all that apply.) A. For maximum support when walking, the patient places the cane forward 15 to 25 cm (6 to 10 inches), keeping body weight on both legs. The weaker leg is moved forward to the cane so body weight is divided between the cane and the stronger leg. C B. A person’s cane length is equal to the distance between the elbow and the floor. C. Canes provide less support than a walker and are less stable. D. The patient needs to learn that two points of support such as both feet or one foot and the cane need to be present at all times.

A. For maximum support when walking, the patient places the cane forward 15 to 25 cm (6 to 10 inches), keeping body weight on both legs. The weaker leg is moved forward to the cane so body weight is divided between the cane and the stronger leg. C. Canes provide less support than a walker and are less stable. D. The patient needs to learn that two points of support such as both feet or one foot and the cane need to be present at all times.

A patient is experiencing some problems with joint stability. The doctor has prescribed crutches for the patient to use while still being allowed to bear weight on both legs. Which of the following gaits should the patient be taught to use? A. Four-point B. Three-point C. Two-point D. Swing-through

A. Four-point

A 56-year-old patient with diabetes admitted for community acquired pneumonia has a temperature of 38.2°C (100.8°F) via the temporal artery. Which additional assessment data are needed in planning interventions for the patient's infection? (Select all that apply.) A. Heart rate B. Presence of diaphoresis C. Smoking history D. Respiratory rate E. Recent bowel movement F. Blood pressure in right arm G. Patient’s normal temperature H. Blood pressure in distal extremity

A. Heart rate Correct B. Presence of diaphoresis Correct D. Respiratory rate Correct G. Patient’s normal temperature Correct You need to determine the patient's usual temperature to evaluate the degree of temperature elevation. Heart rate and respiratory rate increase with temperature. The presence of diaphoresis may contribute to fluid volume deficit from hyperthermia.

A new graduate nurse is being mentored by a more experienced nurse. They are discussing the ways nurses need to remain active professionally. Which of the statements below indicates the new graduate understands ways to remain involved professionally? (Select all that apply.) A. I am thinking about joining the health committee at my church. B. I need to read newspapers, watch news broadcasts, and search the Internet for information related to health. C.I will join nursing committees at the hospital after I have several years of experience and better understand the issues affecting nursing. D. Nurses do not have very much voice in legislation in Washington, DC, because of the shortage of nurses.

A. I am thinking about joining the health committee at my church.; Correct B. I need to read newspapers, watch news broadcasts, and search the Internet for information related to health.; Correct Nurses need to be actively involved in their community and be aware of current issues in health care. Staying abreast of current news and public opinion through the media is essential. Nurses need to join nursing committees at all levels of their career, not just when they have experience. Nurses have a powerful voice in the legislature.

The nurse recognizes that the older adult's progressive loss of total bone mass and tendency to take smaller steps with feet kept closer together will most likely: A. Increase the patient's risk for falls and injuries. B. Result in less stress on the patient's joints. C. Decrease the amount of work required for patient movement. D. Allow for mobility in spite of the aging effects on the patient's joints.

A. Increase the patient's risk for falls and injuries.

The nurse is planning to teach the student nurse how to assess the hydration status of an older adult. Which techniques are appropriate for this situation? (Select all that apply.) A. Inspect the lips and mucous membranes to determine if they are moist. B. Pinch the skin on the back of the hand to see if the skin tents. C. Check the patient’s pulse and blood pressure. D. Weigh the patient daily.

A. Inspect the lips and mucous membranes to determine if they are moist. Correct C. Check the patient’s pulse and blood pressure. Correct D. Weigh the patient daily. Correct By assessing for moisture of the mucous membranes and lips, the nurse can quickly evaluate the patient's hydration status. Weighing a patient shows increases of fluid volume from day to day that could result from cardiac problems. This provides useful information about fluid status over time. Blood pressure can indicate fluid status, but be aware it also can be related to other diseases. Skin on older individuals loses its elasticity, and assessing skin on the dorsum of the hand provides inaccurate data regarding skin turgor.

Which of the following strategies focus on improving nurse-physician collaborative practice? (Select all that apply.) A. Inviting the physician to attend the practice council meeting B. Participating in physician morning rounds C. Placing physician photos and names in unit newsletter D. Contacting physician promptly to discuss patient problems E. Providing a list of physician contact numbers to all staff nurses

A. Inviting the physician to attend the practice council meeting Correct B. Participating in physician morning rounds Correct D. Contacting physician promptly to discuss patient problems Correct Inviting the physician to attend the practice council meeting, participating in physician morning rounds, and contacting the physician promptly to discuss patient problems improve nurse-physician collaboration by focusing on strategies that are related to professional practice. Collaboration is a process whereby different perspectives are synthesized to better understand complex problems and an outcome that is a shared solution that could not have been accomplished by a single person or organization. Placing physician photos and names in a newsletter and providing physician contact numbers to all staff nurses provide information to the nursing staff and help them identify and contact providers but are not focused specifically on nurses working with physicians to provide patient care.

A staff nurse is talking with the nursing supervisor about the stress that she feels on the job. Which of the following are true about work-related stress? (Select all that apply.) A. Job-related stress can affect the quality of patient care. B. Stress can affect nurses' efficiency and decision making. C. Nurses who talk about feeling stress are unprofessional and should calm down. D. Nurses frequently experience stress with the rapid changes in health care technology. E. Nurses cannot resolve job-related stress.

A. Job-related stress can affect the quality of patient care. Correct B. Stress can affect nurses' efficiency and decision making. Correct D. Nurses frequently experience stress with the rapid changes in health care technology. Correct Nurses frequently experience stress with the rapid changes in health care and when the situation seems out of their personal control. When job stress remains unresolved, patient care and clinical decision making can be affected because the stress is perceived as uncontrolled and all consuming

When teaching a patient about the negative feedback response to stress, the nurse includes which of the following to describe the benefits of this stress response? A. Results in neurophysiological response. B. Reduces body temperature C. Causes a person to be hypervigilant D. Reduces level of consciousness to conserve energy.

A. Negative feedback senses an abnormal state such as lowered body temperature and makes an adaptive response such as shivering to generate body heat to return the body to hormonal homeostasis

Which of the following activities does the nurse delegate to nursing assistive personnel in regard to crutch walking? (Select all that apply.) A. Notify nurse if patient reports pain before, during, or after exercise. B. Notify nurse of patient complaints of increased fatigue, dizziness, light-headedness when obtaining vital signs before and/or after exercise. C. Notify nurse of vital sign values. D. Evaluate the patient’s ability to use crutches properly. E. Prepare the patient for exercise by assisting in dressing and putting on shoes.

A. Notify nurse if patient reports pain before, during, or after exercise. B. Notify nurse of patient complaints of increased fatigue, dizziness, light-headedness when obtaining vital signs before and/or after exercise. C. Notify nurse of vital sign values. E. Prepare the patient for exercise by assisting in dressing and putting on shoes.

Which one of the following instructions is crucial for the nurse to give to both family members and the patient who is about to be started on a patient-controlled analgesia (PCA) of morphine? A. Only the patient should push the button. B. Do not use the PCA until the pain is severe. C. The PCA prevents overdoses from occurring. D. Notify the nurse when the button is pushed. o;p[ ]|-

A. Only the patient should push the button. Correct Patient preparation and teaching are critical to the safe and effective use of PCA devices. Patients need to understand PCA and be physically able to locate and press the button to deliver the dose. Be sure to instruct family members not to "push the button" for the patient.

The nurse collects the supplies for the dressing change for the patient in bed 1 and signs out the capillary blood glucose monitoring equipment to test the glucose of the patient in bed 2 before walking down the hall to the room. The nurse is displaying: A. Organizational skills. B. Use of resources. C. Priority setting. D. Clinical decision making.

A. Organizational skills. Correct

Which of the following patients is at most risk for tachypnea? (Select all that apply.) A. Patient just admitted with four rib fractures B. Woman who is 9 months’ pregnant C. Adult who has consumed alcoholic beverages D. Adolescent awaking from sleep

A. Patient just admitted with four rib fractures Correct B. Woman who is 9 months’ pregnant Correct Rib fractures would cause splinting and pain to increase respiratory rate. Pregnancy impedes diaphragmatic excursion, causing shallow, frequent breaths.

Before transferring a patient from the bed to a stretcher, which assessment data does the nurse need to gather? (Select all that apply.) A. Patient’s weight B. Patient’s level of cooperation C. Patient’s ability to assist D. Presence of medical equipment E. 24-hour calorie intake

A. Patient’s weight Correct B. Patient’s level of cooperation Correct C. Patient’s ability to assist Correct D. Presence of medical equipment Correct By assessing the patient thoroughly you make the correct decision concerning your ability to manage him or her safely, the need for additional personnel, the patient's ability or inability to assist you with the transfer, and the proper equipment to use for the transfer. The calorie intake for the past 24 hours does not affect safe transfer.

A patient is discharged after an exacerbation of chronic obstructive pulmonary disease (COPD). She states, "I'm afraid to go to pulmonary rehabilitation." What is your best response? A. Pulmonary rehabilitation provides a safe environment for monitoring your progress. B. You have to participate or you will be back in the hospital. C. Tell me more about your concerns with going to pulmonary rehabilitation. D. The staff at our pulmonary rehabilitation facility are professionals and will not cause you any harm.

A. Pulmonary rehabilitation provides a safe environment for monitoring your progress.

A crisis intervention nurse is working with a mother whose Down syndrome child has been hospitalized with pneumonia and who has lost her child's disability payment while the child is hospitalized. The mother worries that her daughter will fall behind in special-school classes during hospitalization. Which strategies are effective in helping this mother cope with these stressors? (Select all that apply.) A. Referral to social service process reestablishing the child's disability payment B. Sending the child home in 72 hours and having the child return to school C. Coordinating hospital-based and home-based schooling with the child's teacher D. Teaching the mother signs and symptoms of a respiratory tract infection E. Telling the mother that the stress will decrease in 6 weeks when everything is back to normal

A. Referral to social service process reestablishing the child's disability payment Correct C. Coordinating hospital-based and home-based schooling with the child's teacher Correct D. Teaching the mother signs and symptoms of a respiratory tract infection Correct The stressors for this parent are her child's illness, missing school, and loss of disability payments. Obtaining resources to resolve these stressors will reduce the mother's stress load and allow her to focus on helping her child improve and preventing another respiratory tract infection. Discharging the child in 72 hours with a return to school may not be best for the child's physical condition and may make the situation worse. Giving the mom a 6-week time frame is unrealistic because everyone's time frame is different. The mom may also need to adjust to a "new normal."

Your nursing manager distributes biweekly newsletters of ongoing unit or health care agency activities and posts minutes of committee meetings on a bulletin board in the staff break room. This is an example of: A. Staff communication. B. Problem-solving committees. C. Interdisciplinary collaboration. D. Nurse-physician collaborative practice.

A. Staff communication. Correct A manager's greatest challenge, especially if a work group is large, is communication with staff. Posting minutes in an accessible place for all staff ensures that all staff members receive the same message: the correct message. Minutes of committee meetings are usually in an accessible location for all staff to read.

Which of the following actions, if performed by a registered nurse, would result in both criminal and administrative law sanctions against the nurse? (Select all that apply.) A. Taking or selling controlled substances B. Refusing to provide health care information to a patients child C. Reporting suspected abuse and neglect of children D. Applying physical restraints without a written physician's order

A. Taking or selling controlled substances Correct D. Applying physical restraints without a written physicians order Correct The inappropriate use of controlled substances is prohibited by every Nurse Practice Act. A physical restraint can be applied only on the written order of a health care provider based on Joint Commission and Medicare guidelines.

Which of the following actions, if performed by a registered nurse, would result in both criminal and administrative law sanctions against the nurse? (Select all that apply.) A. Taking or selling controlled substances B. Refusing to provide health care information to a patient's child C. Reporting suspected abuse and neglect of children D. Applying physical restraints without a written physician's order Correct E. Completing an occurrence report on the unit

A. Taking or selling controlled substances Correct D. Applying physical restraints without a written physician's order Correct The inappropriate use of controlled substances is prohibited by every Nurse Practice Act. A physical restraint can be applied only on the written order of a health care provider on the basis of The Joint Commission and Medicare guidelines.

The nurse is planning a staff education conference about abdominal assessment. Which point is important for the nurse to include? A. The aorta can be felt using deep palpation in the upper abdomen near the midline. Correct B. The patient should be sitting to best determine the contour and shape of the abdomen. C. Always wear gloves when palpating the skin on the patient's abdomen. D. Avoid palpating the abdomen if the patient reports any discomfort or feelings of fullness.

A. The aorta can be felt using deep palpation in the upper abdomen near the midline. Correct Complete abdominal assessment includes inspection, followed by auscultation, palpation, and percussion (if warranted). Anatomically the aorta is located in the upper abdomen and can be palpated on an average-sized patient. The assessment should be performed when the patient is supine so all assessment techniques can be included. Unless there is an open wound or other abdominal drainage, the aorta should be palpated without gloves to be able to assess skin texture, temperature, and any unusual pulsations. Palpation should be performed routinely, but leave areas of discomfort or pain until last.

A postoperative patient is currently asleep. Therefore the nurse knows that: A. The sedative administered may have helped him sleep, but assessment of pain is still needed. B. The intravenous (IV) pain medication is effectively relieving his pain. C. Pain assessment is not necessary. D. The patient can be switched

A. The sedative administered may have helped him sleep, but assessment of pain is still needed. Correct Sedatives, antianxiety agents, and muscle relaxants have no analgesic effect; however, they can cause drowsiness and impaired coordination, judgment, and mental alertness and contribute to respiratory depression. It is important to avoid attributing these adverse effects solely to the opioid. You need to conduct a thorough reassessment.

1. Successful ethical discussion depends on people who have a clear sense of personal values. When a group of people share many of the same values, it may be possible to refer for guidance to philosophical principals of utilitarianism. This philosophy proposes which of the following? A. The value of something is determined by its usefulness to society. B. People's values are determined by religious leaders. C. The decision to perform a liver transplant depends on a measure of the moral life that the patient has led so far. D. The best way to determine the solution to an ethical dilemma is to refer the case to the attending physician or health care provider.

A. The value of something is determined by its usefulness to society.

Successful ethical discussion depends on people who have a clear sense of personal values. When a group of people share many of the same values, it may be possible to refer for guidance to philosophical principals of utilitarianism. This philosophy proposes which of the following? A. The value of something is determined by its usefulness to society. B. People's values are determined by religious leaders. C. The decision to perform a liver transplant depends on a measure of the moral life that the patient has led so far. D. The best way to determine the solution to an ethical dilemma is to refer the case to the attending physician or health care provider.`

A. The value of something is determined by its usefulness to society. Correct Utilitarianism specifically refers to the greatest good for the greatest number of people, whereas goodness is determined primarily by usefulness. The concept is easier to apply in a community where shared values allow for agreement about a definition of usefulness.

The effects of immobility on the cardiac system include which of the following? (Select all that apply.) A. Thrombus formation B. Increased cardiac workload C. Weak peripheral pulses D. Irregular heartbeat E. Orthostatic hypotension

A. Thrombus formation Correct B. Increased cardiac workload Correct\ E. Orthostatic hypotension Correct

Resolution of an ethical dilemma involves discussion with the patient, the patient's family, and participants from all health care disciplines. Which of the following describes the role of the nurse in the resolution of ethical dilemmas? A. To articulate his or her unique point of view, including knowledge based on clinical and psychosocial observations B. To await new clinical orders from the physician C. To limit discussions about ethical principals D. To allow the patient and the physician to resolve the dilemma without regard to personally held values or opinions regarding the ethical issues

A. To articulate his or her unique point of view, including knowledge based on clinical and psychosocial observations

Resolution of an ethical dilemma involves discussion with the patient, the patient's family, and participants from all health care disciplines. Which of the following describes the role of the nurse in the resolution of ethical dilemmas? A. To articulate his or her unique point of view, including knowledge based on clinical and psychosocial observations Correct B. To await new clinical orders from the physician C. To limit discussions about ethical principals D. To allow the patient and the physician to resolve the dilemma without regard to personally held values or opinions regarding the ethical issues

A. To articulate his or her unique point of view, including knowledge based on clinical and psychosocial observations Correct Nursing plays a unique and critical role in the resolution of difficult ethical situations. The nurse is often able to contribute information not available to others on the team, the result of the special relationship that nurses build with patients. In providing this information, it is important to remain aware of one's own values and how they may differ from those of the patient and others on the health care team.

Resolution of an ethical dilemma involves discussion with the patient, the patient's family, and participants from all health care disciplines. Which of the following best describes the role of the nurse in the resolution of ethical dilemmas? A. To articulate the nurse's unique point of view, including knowledge based on clinical and psychosocial observations. B. To study the literature on current research about the possible clinical interventions available for the patient in question. C. To hold a point of view but realize that respect for the authority of administrators and physicians takes precedence over personal opinion. D. To allow the patient and the physician to resolve the dilemma on the basis of ethical principles without regard to personally held values or opinions.

A. To articulate the nurse's unique point of view, including knowledge based on clinical and psychosocial observations. Correct A nurse's point of view is essential to full discussion of ethical issues because of the nature of the relationship that nurses develop with patients and the intensity and intimacy of contact with the patient and family.

While caring for a patient with cancer pain, the nurse knows that the World Health Organization (WHO) analgesic ladder recommends: A. Transitioning use of adjuvants with nonsteroidal antiinfl ammatory drugs (NSAIDs) to opioids. B. Using acetaminophen for refractory pain. C. Limiting the use of opioids because of the likelihood of side effects. D. Avoiding total sedation, regardless of how severe the pain is.

A. Transitioning use of adjuvants with nonsteroidal antiinfl ammatory drugs (NSAIDs) to opioids. Correct The WHO analgesic ladder transitions from the use of nonopioids (NSAIDS) with or without adjuvants to opioids with or without adjuvants. Acetaminophen is recommended for lesser levels of pain. Side effects related to the use of opioids may be unavoidable but are treatable. Treatment for severe pain may result in some level of sedation.

A patient recovering from bilateral knee replacements is prescribed bilateral partial weight bearing. You reinforce crutch walking knowing that which of the following crutch gaits is most appropriate for this patient? A. Two-point gait B. Three-point gait C. Four-point gait D. Swing-through gait

A. Two-point gait

The nurse teaches a patient about cranial nerves to help explain why the patient's right side of the mouth droops instead of moving up into a smile. What nerve does the nurse explain to the patient? A. VII — Facial B. V — Trigeminal C. XII — Hypoglossal D. XI— Spinal accessory

A. VII — Facial Correct The facial nerve innervates the sensory and motor functions of the face above the brow, the cheeks, and the chin and controls face symmetry and smi

During the assessment interview of an older woman who is recently widowed, the nurse suspects that this woman is experiencing a developmental crisis. Which of the following questions provide information about the impact of this crisis? (Select all that apply.) A. With whom do you talk on a routine basis? B. What do you do when you feel lonely? C. How is having diabetes affecting your life? D. I know this must be hard for you. Let me tell you what might help. E. Do you have any changes in lifestyle habits: sleeping, eating, smoking, and drinking?

A. With whom do you talk on a routine basis? Correct B. What do you do when you feel lonely? Correct E. Do you have any changes in lifestyle habits: sleeping, eating, smoking, and drinking? Correct A developmental crisis occurs as a person moves through the stages of life, including widowhood. It is important to gather information about how this crisis affects her interactions, coping with loneliness, and any changes in lifestyle habits. Although stress can affect diabetes, there is nothing in this situation that states that the woman has diabetes. Saying, "I know this must be hard for you. Let me tell you what might help" is unacceptable, because the whole purpose of assessment is to gather data and let the patient tell his or her story.

Answering for one's actions

Accountability

Refers to the support of a particular cause

Advocacy

Match the examples with the professional nursing code of ethics: Term Definition Advocacy Responsibility Accountability Confidentiality Definitions 1. You see an open medical record on the computer and close it so no one else can read the record without proper access. 2. You administer a once-a-day cardiac medication at the wrong time, but nobody sees it. However, you contact the primary care provider and your head nurse and follow agency procedure. 3. A patient at the end of life wants to go home to die, but the family wants every care possible. The nurse contacts the primary care provider about the patient's request. 4. You tell your patient that you will return in 30 minutes to give him his next pain medication.

Advocacy - 3. A patient at the end of life wants to go home to die, but the family wants every care possible. The nurse contacts the primary care provider about the patient's request. Responsibility - 4. You tell your patient that you will return in 30 minutes to give him his next pain medication. Accountability -2. You administer a once-a-day cardiac medication at the wrong time, but nobody sees it. However, you contact the primary care provider and your head nurse and follow agency procedure. Confidentiality -1. You see an open medical record on the computer and close it so no one else can read the record without proper access.

The term body alignment of the joints, tendons, ligaments, and muscles while standing, sitting, and lying

Alignment and Balance

The body alignment of the patient in the tripod position includes the following: (Select all that apply.) A. An erect head and neck B. Straight vertebrae C. Extended hips and knees D. Axillae resting on the crutch pads E. Bent knees and hips

An erect head and neck B. Straight vertebrae C. Extended hips and knees

Refers to freedom from external control .

Autonomy Similarly in health care , respect for autonomy refers to the commitment to include patients in decision about all aspects of care as a way of acknowledgment and protecting the patients independence

Which of the following statements made by an older adult reflects the best understanding of the need to exercise regardless of age? A. "You are never too old to begin an exercise program." B. "My grandaughter and I walk together around the high school track 3 times a week." C. "I purchased a subscription to a runner's magazine for my grandson for Christmas." D. "When I was a child, I exercised more than I see kids doing today."

B. "My grandaughter and I walk together around the high school track 3 times a week."

You ask another nurse how to collect a laboratory specimen. The nurse raises her eyebrows and asks, "Why don't you figure it out?" What would be the best response? A. Say nothing and walk away. Find a different nurse to help you. B. "When you brush me off like that, it takes me even longer to do my job." C. "Why do you always put me down like that?" D. "I guess I just enjoy having you make fun of me."

B. "When you brush me off like that, it takes me even longer to do my job."

Which of the following statements would be most likely to block communication? A. "You look kind of tired today." B. "Why do you always put so much salt on your food?" C. "It sounds like this has been a hard time for you." D. "If you use your oxygen when you walk, you may be able to walk farther."

B. "Why do you always put so much salt on your food?"

Ethical dilemmas often arise over a conflict of opinion. Reliance on a predictable series of steps can help people in conflict find common ground. All of the following actions can help resolve conflict. What is the best order of these actions in order to promote the resolution of an ethical dilemma? 1. List the actions that could be taken to resolve the dilemma. 2. Agree on a statement of the problem or dilemma that you are trying to resolve. 3. Agree on a plan to evaluate the action over time. 4. Gather all relevant information regarding the clinical, social, and spiritual aspects of the dilemma. 5. Take time to clarify values and distinguish between facts and opinions—your own and those of others involved. 6. Negotiate a plan. A. 4, 5, 2, 6, 1, 3 B. 4, 5, 2, 1, 6, 3 C. 5, 4, 2, 1, 3, 6 D. 4, 5, 1, 2, 3, 6

B. 4, 5, 2, 1, 6, 3 Correct This is the correct order to determine the dilemma and influencing factors. This process provides opportunities for the nurse and health care team to reflect on personal values and then identify the exact nature of the ethical problem, design a plan, and evaluate the success of the plan.

The patient at greatest risk for developing multiple adverse effects of immobility is a: A. 1-year-old child with a hernia repair. B. 80-year-old woman who has suffered a hemorrhagic cerebrovascular accident (CVA). C. 51-year-old woman following a thyroidectomy. D. 38-year-old woman undergoing a hysterectomy.

B. 80-year-old woman who has suffered a hemorrhagic cerebrovascular accident (CVA). Correct The older the patient and the greater the period of immobility, which can be significant following a hemorrhagic stroke, the greater is the number of systems that can be affected by the immobility.

During the assessment interview of an older woman experiencing a developmental crisis, the nurse asks which of the following questions? A. How is this flood affecting your life? B. Since your husband has died, what have you been doing in the evening when you feel lonely? C. How is having diabetes affecting your life? D. I know this must be hard for you. Let me tell you what might help.

B. A developmental crisis occurs as a person moves through life's stages, including widowhood.

The nurse notes that an advance directive is on a patient's medical record. Which statement represents the best description of an advance directive guideline the nurse will follow? A. A living will allows an appointed person to make health care decisions when the patient is in an incapacitated state. B. A living will is invoked only when the patient has a terminal condition or is in a persistent vegetative state. C. The patient cannot make changes in the advance directive once admitted to the hospital. D. A durable power of attorney for health care is invoked only when the patient has a terminal condition or is in a persistent vegetative state.

B. A living will is invoked only when the patient has a terminal condition or is in a persistent vegetative state. Correct A living will does not assign another individual to make decisions for the patient. A durable power of attorney for health care is active when the patient is incapacitated or cognitively impaired. A cognitively intact patient may change an advance directive at any time.

A nurse notes that an advance directive is on a patient's medical record. Which statement represents the best description of an advance directive guideline that the nurse will follow? A. A living will allows an appointed person to make health care decisions when the patient is in an incapacitated state. B. A living will is invoked only when the patient has a terminal condition or is in a persistent vegetative state. C. The patient cannot make changes in the advance directive once admitted to the hospital. D. A durable power of attorney for health care is invoked only when the patient has a terminal condition or is in a persistent vegetative state.

B. A living will is invoked only when the patient has a terminal condition or is in a persistent vegetative state. Correct A living will does not assign another individual to make decisions for the patient. A durable power of attorney for health care is active when the patient is incapacitated or cognitively impaired. A cognitively intact patient may change an advance directive at any time.

Which is the correct gait when a patient is ascending stairs on crutches? A. A modified two-point gait (The affected leg is advanced between the crutches to the stairs.) B. A modified three-point gait (The unaffected leg is advanced between the crutches to the stairs.) C. A swing-through gait D. A modified four-point gait. (Both legs advance between the crutches to the stairs.)

B. A modified three-point gait (The unaffected leg is advanced between the crutches to the stairs.)

Which is the correct gait when a patient is ascending stairs on crutches? A. A modified two-point gait. The affected leg is advanced between the crutches to the stairs. B. A modified three-point gait. The unaffected leg is advanced between the crutches to the stairs. C. A swing-through gait. D. A modified four-point gait. Both legs advance between the crutches to the stairs.

B. A modified three-point gait. The unaffected leg is advanced between the crutches to the stairs.

A crisis intervention nurse working with a mother whose Down syndrome child has been hospitalized with pneumonia and who has lost her entitlement check while the child is hospitalized can expect the mother to regain stability after how long? A. After 2 weeks when the child's pneumonia begins to improve B. After 6 weeks when she adjusts to the child's respiratory status and reestablishes the entitlement checks C. After 1 month when the child goes home and the mother gets help from a food pantry D. After 6 months when the child is back in school

B. After 6 weeks when she adjusts to the child's respiratory status and reestablishes the entitlement checks

To prevent complications of immobility, what would be the most effective activity on the first postoperative day for a patient who has had abdominal surgery? A. Turn, cough, and deep breathe every 30 minutes while awake B. Ambulate patient to chair in the hall C. Passive range of motion 4 times a day D. Immobility is not a concern the first postoperative day

B. Ambulate patient to chair in the hall Correct

The nurses on the unit developed a system for self-scheduling of work shifts. This is an example of: A. Responsibility. B. Autonomy. C. Accountability. D. Authority.

B. Autonomy. Correct Autonomy is freedom of choice and responsibility for the choices. One type of autonomy for nurses is work autonomy. In work autonomy the nurse makes independent decisions about the work of the unit such as scheduling or unit governance.

A child's immunization may cause discomfort during administration, but the benefits of protection from disease, both for the individual and society, outweigh the temporary discomforts. Which principle is involved in this situation? A. Fidelity B. Beneficence C. Nonmaleficence D. Respect for autonomy

B. Beneficence

A child's immunization may cause discomfort during administration, but the benefits of protection from disease, both for the individual and society, outweigh the temporary discomforts. Which principle is involved in this situation? A. Fidelity B. Beneficence C. Nonmaleficence D. Respect for autonomy

B. Beneficence Correct The immunization is a clear effort to provide benefit. Beneficence refers to "doing good." Fidelity refers more to keeping promises. Nonmaleficence refers to the commitment to avoid harm. Respect for autonomy refers to the commitment to include patients in the decision-making process regarding health care plans.

The nurse states, "When you tell me that you're having a hard time living up to expectations, are you talking about your family's expectations?" The nurse is using which therapeutic communication technique? A. Providing information B. Clarifying C. Focusing D. Paraphrasing

B. Clarifying

Mrs. Jones states that she gets anxious when she thinks about giving herself insulin. How do you use your understanding of intrapersonal communication to help with this? A. Provide her the opportunity to practice drawing up insulin B. Coach her to give herself positive messages about her ability to do this C. Bring her written material that clearly describes the steps of insulin administration D. Use therapeutic communication to help her express her feeling about giving herself an injection

B. Coach her to give herself positive messages about her ability to do this

While auscultating the adult patient's lungs, the nurse hears loud, bubbly sounds during inspiration that did not disappear after the patient coughed. Which finding should the nurse document from the lung assessment? A. Rhonchi B. Coarse crackles C. Sibilant wheeze D. Pleural friction rub

B. Coarse crackles Correct Coarse crackles are loud, bubbly sounds heard during inspiration that are not cleared with coughing. This is because of random, sudden reinflation of groups of alveoli and the disruption of the passage of air through the small

A precise definition for the word quality is difficult to articulate when it comes to quality of life. Why? (Select all that apply.) A. Quality of life is measured by potential income, and average income varies in different regions of the country. B. Community values are subject to change, and communities influence definitions of “quality.” C. Individual experiences influence perceptions of quality in potentially different ways, making consensus difficult. D. Placing measurable value on elusive elements such as cognitive skills, ability to perform meaningful work, and relationship to family is challenging.

B. Community values are subject to change, and communities influence definitions of “quality.” C. Individual experiences influence perceptions of quality in potentially different ways, making consensus difficult. D. Placing measurable value on elusive elements such as cognitive skills, ability to perform meaningful work, and relationship to family is challenging.

A precise definition for the word quality is difficult to articulate when it comes to quality of life. Why? (Select all that apply.) A. Quality of life is measured by potential income, and average income varies in different regions of the country. B. Community values are subject to change, and communities influence definitions of quality C. Individual experiences influence perceptions of quality in potentially different ways, making consensus difficult. D. Placing measurable value on elusive elements such as cognitive skills, ability to perform meaningful work, and relationship to family is challenging.

B. Community values are subject to change, and communities influence definitions of quality Correct C. Individual experiences influence perceptions of quality in potentially different ways, making consensus difficult. Correct D. Placing measurable value on elusive elements such as cognitive skills, ability to perform meaningful work, and relationship to family is challenging. Correct Definitions of quality depend on individual experience and value integrated into community standards, but value refers to elements beyond monetary value.

It can be difficult to agree on a common definition of the word quality when it comes to quality of life. Why? (Select all that apply.) A. Average income varies in different regions of the country. B. Community values influence definitions of quality, and they are subject to change over time. C. Individual experiences influence perceptions of quality in different ways, making consensus difficult. D. The value of elements such as cognitive skills, ability to perform meaningful work, and relationship to family is difficult to quantify using objective measures. E. Statistical analysis is difficult to apply when the outcome cannot be quantified. F. Whether or not a person has a job is an objective measure, but it does not play a role in understanding quality of life.

B. Community values influence definitions of quality, and they are subject to change over time. Correct C. Individual experiences influence perceptions of quality in different ways, making consensus difficult. Correct D. The value of elements such as cognitive skills, ability to perform meaningful work, and relationship to family is difficult to quantify using objective measures. Correct E. Statistical analysis is difficult to apply when the outcome cannot be quantified. Correct A person's average income and whether the person is employed are incorrect answers because income level is not necessarily a determining factor in measuring quality of life, but the ability to do meaningful work usually does influence the definition.

Which of the following signs or symptoms in an opioid-naïve patient is of greatest concern to the nurse when assessing the patient 1 hour after administering an opioid? A. Oxygen saturation of 95% B. Difficulty arousing the patient C. Respiratory rate of 10 breaths/min D. Pain intensity rating of 5 on a scale of 0 to 10

B. Difficulty arousing the patient Correct Opioid-naive patients may develop a rare adverse effect of respiratory depression, and sedation always occurs before respiratory depression.

After the 0700 shift report the registered nurse (RN) delegates three tasks to the nursing assistant. At 1300 the RN tells the nursing assistant that he would like to talk to her about the first task that was delegated, which was walking the patient, Mrs. Taylor, earlier that morning. The RN says, "You did a good job walking Mrs. Taylor by 0930. I saw that you recorded her pulse before and after the walk. I saw that Mrs. Taylor walked in the hallway barefoot. For safety, the next time you walk a patient, you need to make sure that the patient wears slippers or shoes. Please walk Mrs. Taylor again by 1500." Which characteristics of good feedback did the RN use when talking to the nursing assistant? (Select all that apply.) A. Feedback is given immediately. B. Feedback focuses on one issue. C. Feedback offers concrete details. D. Feedback identifies ways to improve. E. Feedback focuses on changeable things. F. Feedback is specific about what is done incorrectly only.

B. Feedback focuses on one issue. Correct C. Feedback offers concrete details. Correct D. Feedback identifies ways to improve. Correct E. Feedback focuses on changeable things. Correct

Ethical dilemmas often arise over a conflict of opinion. What is the critical first step in negotiating the difference of opinion? A. Consult a professional ethicist to ensure that the steps of the process occur in full. B. Gather all relevant information regarding the clinical, social, and spiritual aspects of the dilemma. C. Ensure that the attending physician or health care provider has written an order for an ethics consultation to support the ethics process. D. List the ethical principles that inform the dilemma so negotiations agree on the language of the discussion.

B. Gather all relevant information regarding the clinical, social, and spiritual aspects of the dilemma.

Ethical dilemmas often arise over a conflict of opinion. What is the critical first step in negotiating the difference of opinion? A. Consult a professional ethicist to ensure that the steps of the process occur in full. B. Gather all relevant information regarding the clinical, social, and spiritual aspects of the dilemma. C. Ensure that the attending physician or health care provider has written an order for an ethics consultation to support the ethics process. D. List the ethical principles that inform the dilemma so negotiations agree on the language of the discussion.

B. Gather all relevant information regarding the clinical, social, and spiritual aspects of the dilemma. Correct Before proceeding with discussion about any difficult situation, just as in the nursing process, participants take time to gather all relevant information as insurance for reliability and validity during the discussion.

The application of utilitarianism does not always resolve an ethical dilemma. Which of the following statements best explains why? A. Utilitarianism refers to usefulness and therefore eliminates the need to talk about spiritual values. B. In a diverse community it can be difficult to find agreement on a definition of usefulness, the focus of utilitarianism. C. Even when agreement about a definition of usefulness exists in a community, laws prohibit an application of utilitarianism. D. Difficult ethical decisions cannot be resolved by talking about the usefulness of a procedure.

B. In a diverse community it can be difficult to find agreement on a definition of usefulness, the focus of utilitarianism. Correct In increasingly diverse communities, ideas of usefulness have become equally diverse.

A patient with type 2 diabetes is experiencing a lot of work-related stress and is fearful of losing his job. In addition, his wife is threatening divorce. His blood sugar is elevating, and his doctors want him to attend some stress-management classes. He says, "My blood sugar can't be high because of my work stress." What causes blood glucose to rise during stress? (Select all that apply.) A. Increases in antidiuretic hormone (ADH) B. Increases in cortisol C. Increases in aldosterone D. Increases in adrenocorticotropic hormone (ACTH) E. Increases in epinephrine

B. Increases in cortisol Correct D. Increases in adrenocorticotropic hormone (ACTH) Correct E. Increases in epinephrine Correct With stress the general adaptation syndrome is present. Glucose levels rise because ACTH stimulates cortisol, and gluconeogenesis occurs; the body creates new glucose from nonglucose sources (proteins and fats); cortisol alone increases gluconeogenesis; the sympathetic nervous system causes increased epinephrine, which elevates blood glucose. In the person with diabetes, these physiological responses can cause blood glucose levels to elevate beyond normal. ADH and aldosterone affect sodium and/or water balance and do not affect blood glucose.

The nurse evaluates that the NAP has applied a patient's sequential compression device (SCD) appropriately when which of the following is observed? (Select all that apply.) A. Initial patient measurement is made around the calves B. Inflation pressure averages 40 mm Hg C. Patient's leg placed in SCD sleeve with back of knee aligned with popliteal opening on the sleeve. D. Stockings are removed every 2 hours during application. E. Yellow light indicates SCD device is functioning.

B. Inflation pressure averages 40 mm Hg Correct C. Patient's leg placed in SCD sleeve with back of knee aligned with popliteal opening on the sleeve. Correct

Which is an outcome for a patient diagnosed with osteoporosis? A. Maintain serum level of calcium. B. Maintain independence with activities of daily living (ADLs). C. Reduce supplemental sources of vitamin D. D. Reverse bone loss through dietary manipulation.

B. Maintain independence with activities of daily living (ADLs). Correct The main goal is to maintain independence in ADLs once osteoporosis is diagnosed. It is best to identify individuals at risk and work toward preventing the disease.

The nurse is explaining the case management model to a group of nursing students. Which characteristics best describe the model? (Select all that apply.) A. Case managers provide all patient care. B. Multidisciplinary care plans are used. C. Case managers coordinate discharge planning. D. Staffing is expensive and may not decrease care costs. E. Communication with health care team members is important. F. Model helps to improve patient safety and quality.

B. Multidisciplinary care plans are used. Correct C. Case managers coordinate discharge planning. Correct E. Communication with health care team members is important. Correct F. Model helps to improve patient safety and quality. Correct Case management is a care management approach that coordinates and links health care services to patients and their families while streamlining costs and maintaining quality. Activities listed in answers 2, 3, 5, and 6 are characteristics of the case management model or responsibilities of the case manager. Answers 1 and 4 are not characteristics of the case management model. They are typically seen with the primary care model.

When an ethical dilemma occurs on your unit, can you resolve the dilemma by taking a vote? A. Yes because ethics is essentially a democratic process, with all participants sharing an equal voice B. No because an ethical dilemma involves the resolution of conflicting values and principals rather than simply the identification of what people want to do C. Yes because ethical dilemmas otherwise take up time and energy that is better spent at the bedside performing direct patient care D. No because most ethical dilemmas are resolved by deferring to the medical director of the ethics department

B. No because an ethical dilemma involves the resolution of conflicting values and principals rather than simply the identification of what people want to do

When an ethical dilemma occurs on your unit, can you resolve the dilemma by taking a vote? A. Yes because ethics is essentially a democratic process, with all participants sharing an equal voice B. No because an ethical dilemma involves the resolution of conflicting values and principals rather than simply the identification of what people want to do C. Yes because ethical dilemmas otherwise take up time and energy that is better spent at the bedside performing direct patient care D. No because most ethical dilemmas are resolved by deferring to the medical director of the ethics department

B. No because an ethical dilemma involves the resolution of conflicting values and principals rather than simply the identification of what people want to do Correct Voting about an outcome implies that participants simply express an opinion without regard for negotiating differences. The real goal in processing ethical dilemmas is to resolve differences, not simply to express opinion.

Which group of patients is at most risk for severe injuries related to falls? A. Adolescents B. Older adults C. Toddlers D. Young children

B. Older adults

The nurse is teaching a young female patient to practice good skin health. Which information is important for the nurse to include? A. Avoid sunbathing between 3 PM and 7 PM. B. Oral contraceptives and antiinflammatories make the skin more sensitive to the sun. Correct C. Call the health care provider for the presence of a mole on an arm or leg that appears uniformly brown. D. Wear sunscreen with an SPF of 30 or greater if using a sunlamp or tanning parlor.

B. Oral contraceptives and antiinflammatories make the skin more sensitive to the sun. Correct Some medications such as oral contraceptives or antiinflammatory medications may increase the skin's sensitivity to ultraviolet (UV) rays. Skin self-care and self-evaluation practices include avoiding the sun when UV rays are strongest (10 AM to 4 PM). In addition, good skin practices indicate that skin protection should be used when using a tanning bed or sunlamp. Moles that are uniformly brown are not a cause of concern

A patient on bed rest for several days attempts to walk with assistance. He becomes dizzy and nauseated. His pulse rate jumps from 85 to 110 beats/min. These are most likely symptoms of which of the following? A. Rebound hypertension B. Orthostatic hypotension C. Dysfunctional proprioception. D. Central nervous system rebound hypotension

B. Orthostatic hypotension Correct

While assessing the adult patient's lungs, the nurse identifies the following assessment findings. Which finding should be reported to the health care provider? A. Respiratory rate: 14 B. Pain reported when palpating posterior lower thorax Correct C. Thorax rising and falling symmetrically for right and left lungs D. Vesicular breath sounds heard with auscultation of peripheral lung fields

B. Pain reported when palpating posterior lower thorax Correct Any areas of tenderness or pain over the posterior thorax could indicate injury such as a broken rib or disturbance of the integumentary system. Further palpation should be avoided until more assessment data are collected, either through further health history or diagnostic testing. All other findings are normal.

A child who has been in a house fire comes to the emergency department with her parents. The child and parents are upset and tearful. During the nurse's first assessment for stress the nurse says: A. "Tell me who I can call to help you." B. "Tell me what bothers you the most about this experience." C. "I'll contact someone who can help get you temporary housing." D. "I'll sit with you until other family members can come help you get settled."

B. Physical causes for problems need to be discovered before treatment for psychosocial problems can be initiated.

A nurse is caring for a patient who recently had coronary bypass surgery and now is on the postoperative unit. Which are legal sources of standards of care that the nurse uses to deliver safe health care? (Select all that apply.) A. Information provided by the head nurse B. Policies and procedures of the employing hospital C. State Nurse Practice Act D. Regulations identified in The Joint Commission manual E. The American Nurses Association standards of nursing practice

B. Policies and procedures of the employing hospital Correct C. State Nurse Practice Act Correct D. Regulations identified in The Joint Commission manual Correct E. The American Nurses Association standards of nursing practice Correct All of these sources govern the legal standards of care and are individualized by state and agency. Policies and procedures of employing agencies and standards set by statutes, accrediting agencies, and professional organizations describe the minimum requirements for safe care.

1. A nurse is caring for a patient who recently had coronary bypass surgery. Which are legal sources of standards of care the nurse uses to deliver safe health care? (Select all that apply.) A. Information provided by the head nurse B. Policies and procedures of the employing hospital C. State Nurse Practice Act D. Regulations identified in The Joint Commission;s manual E. The American Nurses Association standards of nursing practice

B. Policies and procedures of the employing hospital Correct C. State Nurse Practice Act Correct D. Regulations identified in The Joint Commission’s manual Correct Commission’s manual Correct E. The American Nurses Association standards of nursing practice Correct Policies and procedures of employing agencies and standards set by statutes, accrediting agencies, and professional organizations describe the minimum requirements for safe care.

Which of the following most motivates a patient to participate in an exercise program? A. Providing a patient with a pamphlet on exercise B. Providing information to the patient when he or she is ready to change behavior C. Explaining the importance of exercise at the time of diagnosis of a chronic disease D. Providing the patient with a booklet with examples of exercises E. Providing the patient with a prescribed exercise program

B. Providing information to the patient when he or she is ready to change behavior

Which of the following best motivates a patient to participate in an exercise program? A. Giving a patient information on exercise B. Providing information to the patient when the patient is ready to change behavior C. Explaining the importance of exercise when a patient is diagnosed with a chronic disease such as diabetes D. Following up with instructions after the health care provider tells a patient to begin an exercise program

B. Providing information to the patient when the patient is ready to change behavior

Your patient has just been told that she has cancer, and she is crying. Which actions facilitate therapeutic communication? (Select all that apply.) A. Turning on the television to her favorite show B. Pulling the curtain to provide privacy C. Offering to discuss information about her condition D. Asking her why she is crying E. Sitting quietly by her bed and hold her hand

B. Pulling the curtain to provide privacy C. Offering to discuss information about her condition E. Sitting quietly by her bed and hold her hand

A patient is admitted for dehydration caused by pneumonia and shortness of breath. He has a history of heart disease and cardiac dysrhythmias. The nursing assistant tells you his admitting vital signs. Which measurement should you reassess? (Select all that apply.) A. Right arm BP: 120/80 B. Radial pulse rate: 72 and irregular C. Temporal temperature: 37.4°C (99.3°F) D. Respiratory rate: 28 E. Oxygen saturation: 99%

B. Radial pulse rate: 72 and irregular Correct D. Respiratory rate: 28 Correct E. Oxygen saturation: 99% Correct An irregular pulse may be the result of a complication of heart disease and requires the assessment of the apical rate. A respiratory rate of 28 is abnormal, yet the oxygen saturation is normal. Both oxygen saturation and respiratory rate would be expected to be outside of the acceptable range.

Which of the following explain how health care reform is an ethical issue? (Select all that apply.) A. Access to care is an issue of beneficence, a fundamental principal in health care ethics. B. Reforms promote the principle of beneficence, a hallmark of health care ethics. C. Purchasing health care insurance may become an obligation rather than a choice, a potential conflict between autonomy and beneficence. D. Lack of access to affordable health care causes harm, and nonmaleficence is a basic principal of health care ethics.

B. Reforms promote the principle of beneficence, a hallmark of health care ethics. C. Purchasing health care insurance may become an obligation rather than a choice, a potential conflict between autonomy and beneficence. D. Lack of access to affordable health care causes harm, and nonmaleficence is a basic principal of health care ethics.

Which of the following explain how health care reform is an ethical issue? (Select all that apply.) A. Access to care is an issue of beneficence, a fundamental principal in health care ethics. B. Reforms promote the principle of beneficence, a hallmark of health care ethics. C. Purchasing health care insurance may become an obligation rather than a choice, a potential conflict between autonomy and beneficence. D. Lack of access to affordable health care causes harm, and nonmaleficence is a basic principal of health care ethics.

B. Reforms promote the principle of beneficence, a hallmark of health care ethics. Correct C. Purchasing health care insurance may become an obligation rather than a choice, a potential conflict between autonomy and beneficence. Correct D. Lack of access to affordable health care causes harm, and nonmaleficence is a basic principal of health care ethics. Correct Since health care reform focuses on the public good, discussions about it inevitably involve reference to all aspects of ethical discourse. Reference to ethical principles helps to shape the discussion, even when individual values differ. Access to care is an issue of justice.

The philosophy sometimes called the ethics of care suggests that ethical dilemmas can best be solved by attention to which of the following? A. Patients B. Relationships C. Ethical principles D. Code of ethics for nurses

B. Relationships

The philosophy sometimes called the ethics of care suggests that ethical dilemmas can best be solved by attention to which of the following? A. Patients B. Relationships C. Ethical principles D. Code of ethics for nurses

B. Relationships Correct The foundation of the ethics of care is its attention to relationships, as distinguished from other more principal based philosophies.

A patient with a history of a stroke that left her confused and unable to communicate returns from interventional radiology following placement of a gastrostomy tube. The health care provider's order reads as follows: "Vicodin 1 tab, per tube, q4 hours, prn." Which action by the nurse is most appropriate? A. No action is required by the nurse because the order is appropriate. B. Request to have the ordered changed to ATC for the first 48 hours. C. Ask for a change of medication to meperidine (Demerol) 50 mg IVP, q3 hours, prn. D. Begin the Vicodin when the patient shows nonverbal symptoms of pain

B. Request to have the ordered changed to ATC for the first 48 hours. Correct The American Pain Society (2003) states that, if you anticipate pain for most of the day, you should consider ATC administration. Insertion of a gastrostomy tube is painful. This patient will most likely experience pain for at least the next 48 hours.

An older adult has limited mobility as a result of a surgical repair of a fracture hip. During assessment you note that the patient cannot tolerate lying flat. Which of the following assessment data support a possible pulmonary problem related to impaired mobility? (Select all that apply.) A. B/P = 128/84 B. Respirations 26 per minute on room air C. HR 114 D. Crackles heard on auscultation E. Pain reported as 3 on scale of 0 to 10 after medication

B. Respirations 26 per minute on room air C. HR 114 D. Crackles heard on auscultation Patients with reduced mobility are at risk for retained pulmonary secretions, and this risk increases in postoperative patients. As a result of retained secretions, the respiratory rate increases. The heart rate also increases because the heart is trying to improve oxygen levels. These symptoms are of concern for older adults because, if left untreated, further complications such as heart failure can occur.

A 55-year-old widowed patient was in a motor vehicle accident and is admitted to a surgical unit after repair of a fractured left arm and left leg. She also has a laceration on her forehead. An intravenous (IV) line is infusing in the right antecubital fossa, and pneumatic compression stockings are on the right lower leg. She is receiving oxygen via a simple face mask. What sites do you instruct the nursing assistant to use for obtaining the patient's blood pressure and temperature? A. Right antecubital and tympanic membrane B. Right popliteal and right axillae C. Left antecubital and oral D. Left popliteal and temporal artery

B. Right popliteal and right axillae Correct The only extremity that does not have a compromised artery to auscultate is the right lower leg after the sequential device is removed. The tympanic membrane and temporal artery are affected by facial surgery and oxygen mask.

A patient is being discharged home on an around-the-clock (ATC) opioid for chronic back pain. Because of this order, the nurse anticipates an order for which class of medication? A. Stool softener B. Stimulant laxative C. H 2 receptor blocker D. Proton pump inhibitor

B. Stimulant laxative Correct Patients usually become tolerant to the side effects of opioids, with the exception of constipation. Routinely administer stimulant laxatives, not simple stool softeners, to prevent and treat constipation.

A nurse is planning care for a patient going to surgery. Who is responsible for informing the patient about the surgery along with possible risks, complications, and benefits? A. Family member B. Surgeon C. Nurse D. Nurse Manager

B. Surgeon Correct The person performing the procedure is responsible for informing the patient about the procedure and its risks, benefits, and possible complications.

A nurse is planning care for a patient going to surgery. Who is responsible for informing the patient about the surgery along with possible risks, complications, and benefits? A. Family member B. Surgeon C. Nurse D. Nurse manager

B. Surgeon Correct The person performing the procedure is responsible for informing the patient about the procedure and its risks, benefits, and possible complications.

The nurse is teaching a patient to prevent heart disease. Which information should the nurse include? (Select all that apply.) A. Limit intake of cholesterol to less than 400 mg/day. B. Talk with your health care provider about taking a daily low dose of aspirin. C. Work with your health care provider to develop a regular exercise program. D. Limit daily intake of fats to less than 25% to 35% of total calories. E. Review strategies to encourage the patient to quit smoking.

B. Talk with your health care provider about taking a daily low dose of aspirin. Correct C. Work with your health care provider to develop a regular exercise program. Correct D. Limit daily intake of fats to less than 25% to 35% of total calories. Correct E. Review strategies to encourage the patient to quit smoking. Correct Teaching about prevention of heart disease focuses on risk factor reduction. Smoking, lack of regular aerobic exercise, and a diet high in fats (which contributes to higher cholesterol levels) are three major risk factors that can be modified. Quitting smoking, regular exercise, and a diet with lower fat intake are preventive measures

A patient of any age can develop a contracture of a joint when: A. The adductors muscles are weakened as a result of immobility. B. The muscle fibers become shortened because of disuse. C. The calcium-to-phosphorus ratio becomes disrupted. D. There is a deficiency in vitamin D.

B. The muscle fibers become shortened because of disuse. Correct

A nurse is sued for negligence due to failure to monitor a patient appropriately after a procedure. Which of the following statements are correct about this lawsuit? (Select all that apply.) A. The nurse does not need any representation. B. The patient must prove injury, damage, or loss occurred. C. The person filing the lawsuit has to show a compensable damage, such as lost wages, occurred. D. The patient must prove that a breach in the prevailing standard of care caused an injury. E. The burden of proof is always the responsibility of the nurse.

B. The patient must prove injury, damage, or loss occurred. Correct C. The person filing the lawsuit has to show a compensable damage, such as lost wages, occurred. Correct D. The patient must prove that a breach in the prevailing standard of care caused an injury. Correct The patient as plaintiff must prove that the defendant nurse had a duty, breached the duty, and because of this breach caused the patient injury or damage.

The patient rates his pain as a 6 on a scale of 0 to 10, with 0 being no pain and 10 being the worst pain. The patient's wife says that he can't be in that much pain since he has been sleeping for 30 minutes. Which is the most accurate resource for assessing the pain? A. The patient's wife is the best resource for determining the level of pain since she has been with him continually for the entire day. B. The patient's report of pain is the best method for assessing the pain. C. The patient's health care provider has the best knowledge of the level of pain that the patient that should be experiencing. D. The nurse is the most experienced at assessing pain.

B. The patient's report of pain is the best method for assessing the pain. Correct A patient's self-report of pain is the single most reliable indicator of the existence and intensity of pain.

A home care nurse is preparing the home for a patient who is discharged to home following a left-sided stroke. The patient is cooperative and can ambulate with a quad-cane. Which of the following must be corrected or removed for the patient's safety? (Select all that apply.) A. The rubber mat in the walk-in shower B. The three-legged stool on wheels in the kitchen C. The braided throw rugs in the entry hallway and between the bedroom and bathroom D. The night-lights in the hallways, bedroom, and bathroom E. The cordless phone next to the patient’s beD

B. The three-legged stool on wheels in the kitchen Correct C. The braided throw rugs in the entry hallway and between the bedroom and bathroom Correct

A new medical resident writes an order for OxyContin SR 10 mg PO q12 hours prn. Which part of the order does the nurse question? A. The drug B. The time interval C. The dose D. The route

B. The time interval Correct Controlled- or extended-release opioid formulations such as OxyContin are available for administration every 8 to 12 hours ATC. Health care providers should not order these long-acting formulations prn.

A patient with a right knee replacement is prescribed no weight bearing on the right leg. You reinforce crutch walking knowing that which of the following crutch gaits is most appropriate for this patient? A. Two-point gait B. Three-point gait C. Four-point gait D. Swing-through gait

B. Three-point gait

The nurse asks the nursing assistant to hold the legs of a female patient during a Foley catheter insertion. This is an example of a nurse displaying: A. Organizational skills. B. Use of resources. C. Time management. D. Evaluation.

B. Use of resources. Correct Appropriate use of resources is an important aspect of clinical care coordination. Resources in this case include members of the health care team. In any setting the patient care is administered more smoothly when staff members work together. The staff needs to ask for assistance, especially when there is an opportunity to make a procedure or activity more comfortable and safer for the patient.

Immobilized patients are at risk for impaired skin integrity. Which of the following interventions would reduce this risk? (Select all that apply.) A. Repositioning patient every 1 to 2 hours while awake B. Using an objective, valid scale to assess patient’s risk for pressure ulcer development C. Using a device to relieve pressure when patient is seated in chair D. Teaching patient how to shift weight at regular intervals while sitting in a chair Correct E. A good rule is: the higher the risk for skin breakdown, the shorter the interval between position changesCorrect

B. Using an objective, valid scale to assess patient’s risk for pressure ulcer development Correct C. Using a device to relieve pressure when patient is seated in chair Correct D. Teaching patient how to shift weight at regular intervals while sitting in a chair Correct E. A good rule is: the higher the risk for skin breakdown, the shorter the interval between position changesCorrect Patients must be repositioned around the clock, not just when they are awake. An objective assessment scale allows the nurse to assess for pressure ulcer risk over time. Once the risk is identified, the assessment tool guides the nurse in selecting appropriate pressure-relief devices. Showing the patient how to reduce his or her risk by shifting pressure is also important. Frequent and meaningful position changes that are in concert with the patient's condition and risk factors are necessary to reduce pressure ulcer developments.

If a nurse decides to withhold a medication because it might further lower the patient's blood pressure the nurse will be practicing the principle of A. responsibility B. accountability C. Competency D. moral behavior

B. accountability

Refers to taking Positive action to help others

Beneficience

The individuals center of gravity is stable

Body Alignment

A term used to describe the coordinated efforts of the muscoskeletal and nervous system

Body Mechanics

A grandfather living in Japan worries about his two young grandsons who disappeared after a tsunami. This is an example of: A. A situational crisis. B. A maturational crisis. C. An adventitious crisis. D. A developmental crisis.

C- An adventitious crisis is a type of crisis resulting from a natural disaster such as a tsunami.

A nursing assistive personnel asks for help to transfer a patient who is 125 lbs (56.8 kg) from the bed to a wheelchair. The patient is unable to help. What is the nurse's best response? A. "As long as we use proper body mechanics, no one will get hurt." B. "The patient only weighs 125 lbs. You don't need my assistance." C. "Call the lift team for additional assistance." D. "The two of us can lift the patient easily."

C. "Call the lift team for additional assistance."

A nursing assistive personnel asks for help to transfer a patient who is 125 pounds (56.8 kg) from the bed to a wheelchair. The patient is unable to assist. What is the nurse's best response? A. "As long as we use proper body mechanics, no one will get hurt." B. "The patient only weighs 125 lb. You don't need my assistance." C. "Call the lift-team for additional assistance." D. "The two of us can easily lift the patient."

C. "Call the lift-team for additional assistance."

The nurse is evaluating the coping success of a patient experiencing stress from being newly diagnosed with multiple sclerosis and psychomotor impairment. Which of the following statements indicate that the patient is beginning to cope with the diagnosis? (Select all that apply.) A. "I'm going to learn to drive a car so I can be more independent." B. "My sister says she feels better when she goes shopping, so I'll go shopping." C. "I'm going to let the occupational therapist assess my home to improve efficiency." D. "I've always felt better when I go for a long walk. I'll do that when I get home." E. "I'm going to attend a support group to learn more about multiple sclerosis."

C. "I'm going to let the occupational therapist assess my home to improve efficiency." Correct Inviting the occupational E. "I'm going to attend a support group to learn more about multiple sclerosis." Inviting the occupational therapist into the patient's home and attending support groups are early indicators that the patient is recognizing some of the challenges of the disease and participating in positive realistic activities to cope with the stressors related to changes in physical functioning. The other options relate to independence and other coping strategies but do not address coping with the specific challenges of the disease.

The nurse plans to assess the patient's memory. Which task should the nurse ask the patient to perform? A. "Tell me where you are." B. "What can you tell me about your illness?" C. "Repeat these numbers back to me: 7...5...8." D. "What does this mean: 'A stitch in time saves nine?'"

C. "Repeat these numbers back to me: 7...5...8." Correct Asking a patient to recall and repeat a series of numbers assesses immediate recall and recent and remote memory. The other tasks assess orientation, knowledge, judgment, and abstract reasoning.

What is the correct order in which elastic stockings should be applied? 1. Identify patient using two identifiers. 2. Smooth any creases or wrinkles. 3. Slide the remainder of the stocking over the patient's heel and up the leg 4. Turn the stocking inside out until heel is reached. 5. Assess the condition of the patient's skin and circulation of the legs. 6. Place toes into foot of the stocking. 7. Use tape measure to measure patient's legs to determine proper stocking size. A. 1, 5, 7, 4, 6, 2, 3 B. 1, 7, 5, 4, 6, 2, 3 C. 1, 5, 7, 4, 6, 3, 2 D. 1, 5, 4, 7, 6, 3, 2

C. 1, 5, 7, 4, 6, 3, 2

The following blood pressures, taken 6 months apart, were recorded from patients screened by the nurse at the assisted living facility. Which patient should be referred to the healthcare provider for hypertension evaluation? A. 120/80, 118/78, 124/82 B. 128/84, 124/86, 128/88 C. 148/82, 148/78, 134/86 D. 154/78, 118/76, 126/84

C. 148/82, 148/78, 134/86 Correct The definition of hypertension requires two elevated blood pressure measurements in a row. All of the other choices describe prehypertension.

When the nurse takes the patient's nursing history, he or she sits: A. Next to the patient. B. 4 to 12 feet from the patient. C. 18 inches to 4 feet from the patient. D. 12 inches to 3 feet from the patient.

C. 18 inches to 4 feet from the patient.

1. The patient for whom you are caring needs a liver transplant to survive. This patient has been out of work for several months and doesn't have health insurance or enough cash. Even though several ethical principles are at work in this case, what are the principles from highest to lowest priority? 1. Accountability: You as the nurse are accountable for the wellbeing of this patient. 2. Respect for autonomy: This patient's autonomy will be violated if he does not receive the liver transplant. 3. Ethics of care: The caring thing that a nurse could provide this patient is resources for a liver transplant. 4. Justice: The greatest question in this situation is how to determine the just distribution of resources. A. 4, 1, 3, 2 B. 2, 4, 3, 1 C. 4, 2, 3, 1 D. 4, 3, 2, 1

C. 4, 2, 3, 1 Correct Understanding the concept of justice helps to enrich the conversation about how to act and lifts the conversation above and beyond the circumstances of the patient. If justice is compromised, respect for autonomy will be hard to maintain. The nurse will be able to care for the patient, but unfortunately her commitment to care does not give her the power to resolve the difficult issue of limited resources. Other concepts are valid but not as relevant to the case.

Which of the following patients is most at risk for tachycardia? A. A healthy professional tennis player B. A patient admitted with hypothermia C. A patient with a fever of 39.4°C (103°F) D. A 90-year-old male taking beta blockers

C. A patient with a fever of 39.4°C (103°F) Correct Patients with a fever have a high heart rate. A healthy athlete has a low heart rate because of conditioning. Hypothermia slows the heart. Beta-blockers reduce heart rate.

While administering medications, the nurse realizes that she has given the wrong dose of medication to a patient. She acts by completing an incident report and notifying the patient's health care provider. The nurse is exercising: A. Authority. B. Responsibility. C. Accountability. D. Decision making.

C. Accountability. Correct Accountability means that nurses are answerable for their actions. It means that they accept the commitment to provide excellent patient care and the responsibility for the outcomes of the actions in providing it. Following institutional policy for reporting errors demonstrates the nurse's commitment to safe patient care.

Which example demonstrates the nurse performing the skill of evaluation? A. The nurse explains the side effects of the new blood pressure medication ordered for the patient. B. The nurse asks the patient to rate pain on a scale of 0 to 10 before administering the pain medication. C. After completing the teaching, the nurse observes the patient draw up and administer an insulin injection. D. The nurse changes the patient's leg ulcer dressing using aseptic technique.

C. After completing the teaching, the nurse observes the patient draw up and administer an insulin injection. Correct Evaluation is one of the most important aspects of clinical care coordination; it involves the determination of patient outcomes. Observing a patient demonstrate teaching is evaluation to ensure that he or she has understood teaching. Answer 2 is not evaluation since it occurs before administering a pain medication. The other options are interventions.

A nurse observes that a patient whose home life is chaotic with intermittent homelessness, a child with spina bifida, and an abusive spouse appears to be experiencing an allostatic load. As a result, the nurse expects to detect which of the following while assessing the patient? A. Posttraumatic stress disorder B. Rising hormone levels C. Chronic illness D. Return of vital signs to normal\

C. An increased allopathic load can result in long-term physiological problems and chronic illness. Posttraumatic stress disorder results from a single traumatic event. Hormone levels rise in the alarm stage. Vital signs return to normal in the resistance stage

After a health care provider has informed a patient that he has colon cancer, the nurse enters the room to find the patient gazing out the window in thought. The nurse's first response is which of the following? A. "Don't be sad. People live with cancer every day." B. "Have you thought about how you are going to tell your family?" C. "Would you like for me to sit down with you for a few minutes so you can talk about this?" D. "I know another patient whose colon cancer was cured by surgery."

C. Ask the patient if he would like you to sit down for a few minutes so he can talk. Providing an open-ended question and an opportunity for the patient to talk allows the nurse to assess the patient's perception of the situation, which is of utmost importance.

A patient has her call bell on and looks frightened when you enter the room. She has been on bed rest for 3 days following a fractured femur. She says, "It hurts when I try to breathe, and I can't catch my breath." Your first action is to: A. Call the health care provider to report this change in condition. B. Give the patient a paper bag to breathe into to decrease her anxiety. C. Assess her vital signs, perform a respiratory assessment, and be prepared to start oxygen. D. Explain that this is normal after such trauma and administer the ordered pain medication.

C. Assess her vital signs, perform a respiratory assessment, and be prepared to start oxygen. These are signs of possible pulmonary emboli, which can be life threatening. You must assess your patient, be prepared to start oxygen, and have someone call the surgeon while you stay with the patient to continue to monitor her status.

A patient presents in the clinic with dizziness and fatigue. The nursing assistant reports a very slow radial pulse of 44. What is your priority intervention? A. Request that the nursing assistant repeat the pulse check B. Call for a stat electrocardiogram (ECG) C. Assess the patient's apical pulse and evidence of a pulse deficit D. Prepare to administer cardiac-stimulating medications

C. Assess the patient's apical pulse and evidence of a pulse deficit Correct Your priority is to assess the patient first. The nurse cannot delegate vital signs to an unstable patient. Therefore first you determine if the patient has a pulse deficit. Calling for a stat electrocardiogram and preparing to administer cardiac-stimulating medications require notification of the health care provider and occur after you assess the patient.

A patient has been transferred to your unit from the respiratory intensive care unit, where he has been for the past 2 weeks recovering from pneumonia. He is receiving oxygen via 4 L nasal cannula. His respiratory rate is 26 breaths/min, and his oxygen saturation is 92%. In planning his care, which information is most helpful in determining your priority nursing interventions? A. Activity order B. Medication list C. Baseline vital signs D. Patient's perception of dyspnea

C. Baseline vital signs Correct Knowledge of baseline vital signs is needed to prioritize care.

The nurse is interviewing a patient in the community clinic and gathers the following information about her: she is intermittently homeless, a single parent with two children who have developmental delays, and is suffering from chronic asthma. She does not laugh or smile, does not volunteer any information, and at times appears close to tears. She has no support system and does not work. She is experiencing an allostatic load. As a result, which of the following would be present during complete patient assessment? (Select all that apply.) A. Posttraumatic stress disorder B. Rising hormone levels C. Chronic illness D. Return of vital signs to normal E. Depression

C. Chronic illness Correct E.Depression Corre ` An increased allopathic load can result in long-term physiological and psychological problems such as chronic illness and depression. Posttraumatic stress disorder results from a single traumatic event. Hormone levels rise in the alarm stage. Vital signs return to normal in the resistance stage.

A home health nurse notices significant bruising on a 2-year-old patient's head, arms, abdomen, and legs. The patient's mother describes the patient's frequent falls. What is the best nursing action for the home health nurse to take? A. Document her findings and treat the patient B. Instruct the mother on safe handling of a 2-year-old child C. Contact a child abuse hotline D. Discuss this story with a colleague

C. Contact a child abuse hotline Correct Nurses are mandated reporters of suspected child abuse. These assessment findings possibly indicate child abuse.

A home health nurse notices significant bruising on a 2-yearold patient's head, arms, abdomen, and legs. The patient's mother describes the patient's frequent falls. What is the best nursing action for the home health nurse to take? A. Document her findings and treat the patient B. Instruct the mother on safe handling of a 2-year-old child C. Contact a child abuse hotline D. Discuss this story with a colleague

C. Contact a child abuse hotline Correct Nurses are mandated reporters of suspected child abuse. These assessment findings possibly indicate child abuse.

Which of the following are physiological outcomes of immobility? A. Increased metabolism B. Reduced cardiac workload C. Decreased lung expansion D. Decreased oxygen demand

C. Decreased lung expansion Correct

Mr. Sakda emigrated from Thailand. When taking care of him, you note that he looks relaxed and smiles but seldom looks at you directly. How do you respond? A. Use therapeutic communication to assess for increased anxiety B. Sit down and position yourself closer so you are at eye level C. Deflect your eyes downward to show respect D. Continue to maintain eye contact

C. Deflect your eyes downward to show respect

You are the night shift nurse and are caring for a newly admitted patient who appears to be confused. The family asks to see the patient's medical record. What is the first nursing action to take? A. Give the family the record B. Give the patient the record C. Discuss the issues that concern the family with them D. Call the nursing supervisor

C. Discuss the issues that concern the family with them Correct Family members do not have the right to private personal health information without the consent of the patient. Establishing a therapeutic relationship with the family and exploring their concerns gives you the information you need to determine how to best address their concerns.

The faith community nurse is teaching the community center women's group about breast cancer risk factors. Which factors does the nurse include? (Select all that apply.) A. First child at the age of 26 years B. Menopause onset at the age of 49 years C. Family history with BRCA1 inherited gene mutation Correct D. Age over 40 years E. Onset of menses before the age of 12 Correct F. Recent use of oral contraceptives

C. Family history with BRCA1 inherited gene mutation Correct D. Age over 40 years Correct E. Onset of menses before the age of 12 Correct F. Recent use of oral contraceptives Correct These are all risk factors for development of breast cancer. Onset of menopause after the age of 55, not at the age of 49, is a risk factor. First child after the age of 30, not birth of a child at 26, is a risk factor.

A patient has undergone surgery for a femoral artery bypass. The surgeon's orders include assessment of dorsalis pedis pulses. The nurse will use which of the following techniques to assess the pulses? (Select all that apply.) A. Place the fingers behind and below the medial malleolus. B. Have the patient slightly flex the knee with the foot resting on the bed. C. Have the patient relax the foot while lying supine. C D. Palpate the groove lateral to the flexor tendon of the wrist. E. Palpate along the top of the foot in a line with the groove between extensor tendons of great and first toes.

C. Have the patient relax the foot while lying supine. Correct E. Palpate along the top of the foot in a line with the groove between extensor tendons of great and first toes. Correct To palpate the dorsalis pedis pulses (located in the feet), ask the patient to relax the foot, and then palpate along the top of the foot in a line with the groove between the extensor tendons of the toe of the great and first toes. Placing fingers behind the medial malleolus is a technique for assessing the posterior tibial pulse. Having a patient slightly flex the knee is a technique for assessing the popliteal artery behind the knee. Palpation of the groove lateral to the flexor tendon of the wrist is the technique to assess the radial artery.

A nurse notes that the health care unit keeps a listing of the patient names at the front desk in easy view for health care providers to more efficiently locate the patient. The nurse talks with the nursing manager because this action is a violation of which act? A. Mental Health Parity Act B. Patient Self-Determination Act (PSDA) C. Health Insurance Portability and Accountability Act (HIPAA) D. Emergency Medical Treatment and Active Labor Act

C. Health Insurance Portability and Accountability Act (HIPAA) Correct The Privacy Rule of HIPAA requires that patient information be protected from unnecessary publication.

A nurse notes that the health care unit keeps a listing of the patient names at the front desk in easy view for health care providers to more efficiently locate the patient. The nurse talks with the nurse manager because this action is a violation of which act? A. Patient Protection and Affordable Care Act (PPACA) B. Patient Self-Determination Act (PSDA) C. Health Insurance Portability and Accountability Act (HIPAA) D. Emergency Medical Treatment and Active Labor Act

C. Health Insurance Portability and Accountability Act (HIPAA) Correct The Privacy Rule of the HIPAA requires that patient information be protected from unnecessary publication.

Which is the best method of negotiating or processing difficult ethical situations? A. Ethical issues arise between dissenting providers and can be best resolved by deference to an independent arbitrator such a chaplain. B. Since ethical issues usually affect policy and procedure, a legal expert is the best consultant to help resolve disputes. C. Institutional ethics committees help to ensure that all participants involved in the ethical dilemma get a fair hearing and an opportunity to express values, feelings, and opinions as a way to find consensus. D. Medical experts are best able to resolve conflicts about outcome predictions.

C. Institutional ethics committees help to ensure that all participants involved in the ethical dilemma get a fair hearing and an opportunity to express values, feelings, and opinions as a way to find consensus.

Which is the best method of negotiating or processing difficult ethical situations? A. Ethical issues arise between dissenting providers and can be best resolved by deference to an independent arbitrator such a chaplain. B. Since ethical issues usually affect policy and procedure, a legal expert is the best consultant to help resolve disputes. C. Institutional ethics committees help to ensure that all participants involved in the ethical dilemma get a fair hearing and an opportunity to express values, feelings, and opinions as a way to find consensus. D. Medical experts are best able to resolve conflicts about outcome predictions.

C. Institutional ethics committees help to ensure that all participants involved in the ethical dilemma get a fair hearing and an opportunity to express values, feelings, and opinions as a way to find consensus. Correct Ethics is ultimately an activity of community, resolved successfully through institutional ethics committees and not easily resolved by deference to a single expert or leader.

The nurse is assisting the patient with coughing and deep-breathing exercises following abdominal surgery. This is which priority nursing need for this patient? A. Low priority B. High priority C. Intermediate priority D. Nonemergency priority

C. Intermediate priority Correct Assisting the patient with cough and deep breathing is an intermediate priority. Intermediate priorities are nonemergency, nonlife-threatening actual or potential needs that the patient and family members are experiencing. Anticipating teaching needs of patients related to a new drug or taking measures to decrease postoperative complications are examples of intermediate priorities.

A patient has been hospitalized for the past 48 hours with a fever of unknown origin. His medical record indicates tympanic temperatures of 38.7°C (101.6°F) (0400), 36.6°C (97.9°F) (0800), 36.9°C (98.4°F) (1200), 37.6°C (99.6°F) (1600), and 38.3°C (100.9°F) (2000). How would you describe this pattern of temperature measurements? A. Usual range of circadian rhythm measurements B. Sustained fever pattern C. Intermittent fever pattern D. Resolving fever pattern

C. Intermittent fever pattern Correct The pattern returns to acceptable levels at least once in 24 hours interspersed with fever spikes.

Which of the following is a principle of proper body mechanics when lifting or carrying objects? A. Keep the knees in a locked position. B. Bend at the waist to maintain a center of gravity. C. Maintain a wide base of support. D. Hold objects away from the body for improved leverage.

C. Maintain a wide base of support.

Which of the following is a principle of proper body mechanics when lifting or carrying objects? (Select all that apply.) A. Keep the knees in a locked position. B. Bend at the waist to maintain a center of gravity. C. Maintain a wide base of support. D. Hold objects away from the body for improved leverage. E. Encourage patient to help as much as possible.

C. Maintain a wide base of support. E. Encourage patient to help as much as possible.

You are caring for an 80-year-old woman, and you ask her a question while you are across the room washing your hands. She does not answer. What is your next action? A. Leave the room quietly since she evidently does not want to be bothered right now B. Repeat the question in a loud voice, speaking very slowly C. Move to her bedside, get her attention, and repeat the question while facing her D. Bring her a communication board so she can express her needs

C. Move to her bedside, get her attention, and repeat the question while facing her

As you are obtaining the oxygen saturation on a 19-year-old college student with severe asthma, you note that she has black nail polish on her nails. You remove the polish from one nail, and she asks you why her nail polish had to be removed. Your best reply is: A. Nail polish attracts microorganisms and contaminates the finger sensor. B. Nail polish increases oxygen saturation. C. Nail polish interferes with sensor function. D. Nail polish creates excessive heat in sensor probe.

C. Nail polish interferes with sensor function. Correct Nail polish reduces light transmission and can alter oxygen saturation measurement

The ANA code of nursing ethics articulates that the nurse "promotes, advocates for, and strives to protect the health, safety, and rights of the patient." This includes the protection of patient privacy. On the basis of this principal, if you participate in a public online social network such as Facebook, could you post images of a patient's x-ray film if you deleted all patient identifiers? A. Yes because patient privacy would not be violated as long as the patient identifiers were removed B. Yes because respect for autonomy implies that you have the autonomy to decide what constitutes privacy C. No because, even though patient identifiers are removed, someone could identify the patient based on other comments that you make online about his or her condition and your place of work D. No because the principal of justice requires you to allocate resources fairly

C. No because, even though patient identifiers are removed, someone could identify the patient based on other

The ANA code of nursing ethics articulates that the nurse "promotes, advocates for, and strives to protect the health, safety, and rights of the patient." This includes the protection of patient privacy. On the basis of this principal, if you participate in a public online social network such as Facebook, could you post images of a patient's x-ray film if you deleted all patient identifiers? A. Yes because patient privacy would not be violated as long as the patient identifiers were removed B. Yes because respect for autonomy implies that you have the autonomy to decide what constitutes privacy C. No because, even though patient identifiers are removed, someone could identify the patient based on other comments that you make online about his or her condition and your place of work D. No because the principal of justice requires you to allocate resources fairly

C. No because, even though patient identifiers are removed, someone could identify the patient based on other comments that you make online about his or her condition and your place of work Correct Respect for patient privacy includes making the most conservative decisions possible regarding disclosure of personal health information.

The ANA code of nursing ethics articulates that the nurse "promotes, advocates for, and strives to protect the health, safety, and rights of the patient." This includes the protection of patient privacy. On the basis of this principle, if you participate in a public online social network such as Facebook, could you post images of a patient's x-ray film if you obscured or deleted all patient identifiers? A. Yes, because patient privacy would not be violated since patient identifiers were removed B. Yes, because respect for autonomy implies that you have the autonomy to decide what constitutes privacy C. No, because, even though patient identifiers are removed, someone could identify the patient on the basis of other comments that you make online about his or her condition and your place of work D. No, because the principle of justice requires you to allocate resources fairly

C. No, because, even though patient identifiers are removed, someone could identify the patient on the basis of other comments that you make online about his or her condition and your place of work Correct Information such as comments and photos on social media is widely distributed and becomes a risk for violation of privacy. People often inadvertently give "clues" or hints to the identity of a person, or people accessing your site could know your actual assignment or put "two and two" together.

The point of the ethical principal to "do no harm" is an agreement to reassure the public that in all ways the health care team not only works to heal patients but agree to do this in the least painful and harmful way possible. Which principle describes this agreement? A. Beneficence B. Accountability C. Nonmaleficence D. Respect for autonomy

C. Nonmaleficence

The point of the ethical principal to "do no harm" is an agreement to reassure the public that in all ways the health care team not only works to heal patients but agree to do this in the least painful and harmful way possible. Which principle describes this agreement? A. Beneficence B. Accountability C. Nonmaleficence D. Respect for autonomy

C. Nonmaleficence Correct Nonmaleficence refers specifically to the concept of avoiding harm. Beneficence refers more to generosity and goodness, accountability to keeping promises, and respect for autonomy to the commitment by providers to include patients in decisions about all aspects of care.

In most ethical dilemmas in health care, the solution to the dilemma requires negotiation among members of the health care team. Why is the nurse's point of view valuable? A. Nurses understand the principle of autonomy to guide respect for patient's self-worth. B. Nurses have a scope of practice that encourages their presence during ethical discussions. C. Nurses develop a relationship to the patient that is unique among all professional health care providers. D. The nurse's code of ethics recommends that a nurse be present at any ethical discussion about patient care.

C. Nurses develop a relationship to the patient that is unique among all professional health care providers.

In most ethical dilemmas in health care, the solution to the dilemma requires negotiation among members of the health care team. Why is the nurse's point of view valuable? A. Nurses understand the principle of autonomy to guide respect for patient's self-worth. B. Nurses have a scope of practice that encourages their presence during ethical discussions. C. Nurses develop a relationship to the patient that is unique among all professional health care providers. D. The nurse's code of ethics recommends that a nurse be present at any ethical discussion about patient care.`

C. Nurses develop a relationship to the patient that is unique among all professional health care providers. Correct None of these options is wrong, but the point of the question is to build confidence and even pride in the value of the special body of knowledge that a nurse acquires about patients, the result of a unique relationship with them.

In most ethical dilemmas in health care, the solution to the dilemma requires negotiation among members of the health care team. Why is the nurse's point of view valuable? A. Nurses understand the principle of autonomy to guide respect for a patient's self-worth. B. Nurses have a scope of practice that encourages their presence during ethical discussions. C. Nurses develop a relationship with the patient that is unique among all professional health care providers. D. The nurse's code of ethics recommends that a nurse be present at any ethical discussion about patient care.

C. Nurses develop a relationship with the patient that is unique among all professional health care providers. Correct A fundamental goal of this chapter is to promote and nurture the value of the nursing voice in ethical discourse.

A patient with limited English proficiency is going to be discharged on new medication. How does the nurse complete the discharge teaching? A. Uses a dictionary to give directions for medication administration B. Explains the directions to the patient's 14-year-old daughter C. Obtains an interpreter to facilitate communication of medication information D. Uses a picture board and visual aids to communicate medication administration information

C. Obtains an interpreter to facilitate communication of medication information

The patient is assessed for range of joint movement. He or she is unable to move the right arm above the shoulder. How should the nurse document this finding? A. Patient was not able to flex arm at shoulder. B. Extension of right arm is limited. C. Patient's abduction of right arm was limited to 100 degrees. D. Internal rotation of right arm is limited to less than 90 degrees.

C. Patient's abduction of right arm was limited to 100 degrees. Correct Abduction of the arm includes raising the arm away from the side and above the shoulder.

When assessing an older adult who is showing symptoms of anxiety, insomnia, anorexia, and mild confusion, one of the first assessments includes which of the following? A. The amount of family support B. A 3-day diet recall C. A thorough physical assessment D. Threats to safety in her home

C. Physical causes for problems need to be discovered before treatment for psychosocial problems can be initiated.

The nurse is teaching a young mother to palpate her 8-year-old child to quickly evaluate if the child has a fever. Which information is important for the nurse to include? A. Place the palm of the hand on the child's back. B. Lightly touch the child's forehead with the fingertips. C. Place the back of your hand against the child's forehead and then on the back of the neck. D. Use the pads of your fingers and press against the child's neck and over the thorax.

C. Place the back of your hand against the child's forehead and then on the back of the neck. Correct Temperature is best evaluated by palpating the skin with the dorsum or back of the hand. It is best to select two areas to compare to allow you to detect a change in body surface temperature.

The nurse received a hand-off report at the change of shift in the conference room from the night shift nurse. The nursing student assigned to the nurse asks to review the medical records of the patients assigned to them. The nurse begins assessing the assigned patients and lists the nursing care information for each patient on each individual patient's message board in the patient rooms. The nurse also lists the patients' medical diagnoses on the message board. Later in the day the nurse discusses the plan of care for a patient who is dying with the patient's family. Which of these actions describes a violation of the Health Insurance Portability and Accountability Act (HIPAA)? A. Discussing patient conditions in the nursing report room at the change of shift B. Allowing nursing students to review patient charts before caring for patients to whom they are assigned C. Posting medical information about the patient on a message board in the patient's room D. Releasing patient information regarding terminal illness to family when the patient has given permission for information to be shared

C. Posting medical information about the patient on a message board in the patient's room Correct Posting the medical condition of a patient on a message board in the patient's room is not necessary for the patient's treatment. Doing so can result in this information being accessed by persons who are not involved in the patient's treatment.

The nurse received a hand-off report at the change of shift in the conference room from the night shift nurse. The nursing student assigned to the nurse asks to review the medical records of the patients assigned to them. The nurse begins assessing the assigned patients and lists the nursing care information for each patient on each individual patient's message board in the patient rooms. The nurse also lists the patients' medical diagnoses on the message board. Later in the day the nurse discusses the plan of care for a patient who is dying with the patient's family. Which of these actions describes a violation of the Health Insurance Portability and Accountability Act (HIPAA)? A. Discussing patient conditions in the nursing report room at the change of shift B. Allowing nursing students to review patient charts before caring for patients to whom they are assigned C. Posting medical information about the patient on a message board in the patient's room D. Releasing patient information regarding terminal illness to family when the patient has given permission for information to be shared

C. Posting medical information about the patient on a message board in the patient's room Correct Posting the medical condition of a patient on a message board in the patient's room is not necessary for the patient's treatment. Doing so can result in this information being accessed by people who are not involved in the patient's treatment.

A patient on prolonged bed rest is at an increased risk to develop this common complication of immobility if preventive measures are not taken: A. Myoclonus B. Pathological fractures C. Pressure ulcers D. Pruritus

C. Pressure ulcers Correct

Musculoskeletal disorders are the most prevalent and debilitating occupational health hazards for nurses. To reduce the risk for these injuries, the American Nurses Association is advocating which of the following? A. Mandate that physical therapists do all patient transfers B. Require adequate staffing levels in health care organizations C. Require the use of assistive equipment and devices D. Require an adequate number of staff to be involved in all patient transfers .

C. Require the use of assistive equipment and devices

The nurse is assessing a patient who returned 1 hour ago from surgery for an abdominal hysterectomy. Which assessment finding would require immediate follow-up? A. Auscultation of an apical heart rate of 76 B. Absence of bowel sounds on abdominal assessment C. Respiratory rate of 8 breaths/min D. Palpation of dorsalis pedis pulses with strength of +2

C. Respiratory rate of 8 breaths/min CorrectI n healthy adults the normal respiratory rates vary from 12 to 20 respirations per minute. A rate of 8 breaths/min is too low and could be caused by anesthesia or opioid sedation effects.

The patient reports having a sore throat, coughing, and sneezing. While performing a focused assessment, which finding supports the patient's reported symptoms related to upper respiratory infection? A. Buccal mucosa is moist and dark pink. B. Respiratory rate is 18, rhythm is even. C. Retropharyngeal lymph nodes are enlarged and firm. D. Inspection with a tongue depressor on the posterior tongue causes gagging.

C. Retropharyngeal lymph nodes are enlarged and firm. Correct The retropharyngeal nodes are located posteriorly to the throat and are enlarged when an infection is located in the throat or pharynx.

When working with an older adult, the nurse remembers to avoid: A. Touching the patient. B. Allowing the patient to reminisce. C. Shifting quickly from subject to subject. D. Asking the patient how he or she feels.

C. Shifting quickly from subject to subject.

How should the patient be positioned to best palpate for lumps or tumors during an examination of the right breast? A. Supine with both arms overhead with palms upward B. Sitting with hands clasped just above the umbilicus C. Supine with the right arm abducted and hand under the head and neck D. Lying on the right side, adducting the right arm on the side of the body

C. Supine with the right arm abducted and hand under the head and neck Correct Lying on the back allows breast tissue to relax; raising the arm over the patient's head causes the breast tissue to flatten, and palpation can more accurately locate any nodules or tumors, especially cancerous tumors that are fixed against the chest wall.

How should the patient be positioned to best palpate for lumps or tumors during an examination of the right breast? A. Supine with both arms overhead with palms upward B. Sitting with hands clasped just above the umbilicus C. Supine with the right arm abducted and hand under the head and neck D. Lying on the right side, adducting the right arm on the side of the body

C. Supine with the right arm abducted and hand under the head and neck Correct The supine position with one arm under the head allows the breast tissue to flatten evenly against the chest wall. This allows for better palpation for lumps or tumors.

A nurse stops to help in an emergency at the scene of an accident. The injured party files a suit, and the nurse's employing institution insurance does not cover the nurse. What would probably cover the nurse in this situation? A. The nurse's automobile insurance B. The nurse's homeowner's insurance C. The Good Samaritan law, which grants immunity from suit if there is no gross negligence D. The Patient Care Partnership, which may grant immunity from suit if the injured party consents

C. The Good Samaritan law, which grants immunity from suit if there is no gross negligence Correct The Good Samaritan law holds the health care provider immune from liability as long as he or she functions within the scope of his or her expertise.

A nurse stops to help in an emergency at the scene of an accident. The injured party files a suit, and the nurse's employing institution insurance does not cover the nurse. What would probably cover the nurse in this situation? A. The nurse's automobile insurance B. The nurse's homeowner's insurance C. The Good Samaritan laws, which grant immunity from suit if there is no gross negligence D. The Patient Care Partnership, which may grant immunity from suit if the injured party consents

C. The Good Samaritan laws, which grant immunity from suit if there is no gross negligence Correct The Good Samaritan law holds the health care provider immune from liability as long as he or she functions within the scope of his or her expertise.

The nurse notices that a patient has received oxycodone/acetaminophen (Percocet) (5/325) two tablets PO every 3 hours for the past 3 days. What concerns the nurse the most? A. The patient's level of pain B. The potential for addiction C. The amount of daily acetaminophen D. The risk for gastrointestinal bleeding

C. The amount of daily acetaminophen Correct The major adverse effect of acetaminophen is hepatotoxicity. The maximum 24-hour dose is 4 g. It is often combined with opioids (e.g., oxycodone [Percocet]) because it reduces the dose of opioid needed to achieve successful pain control.

The nurse has a patient who is short of breath and calls the health care provider using SBAR (Situation-Background-Assessment-Recommendation) to help with the communication. What does the nurse first address? A. The respiratory rate is 28. B. The patient has a history of lung cancer. C. The patient is short of breath. D. He or she requests an order for a breathing treatment.

C. The patient is short of breath.

1. A nurse is instructing a patient who has decreased leg strength on the left side how to use a cane. Which action indicates proper cane use by the patient? A. The patient keeps the cane on the left side of the body. B. The patient slightly leans to one side while walking. C. The patient keeps two points of support on the floor at all times. D. After the patient places the cane forward, he or she then moves the right leg forward to the cane.

C. The patient keeps two points of support on the floor at all times.

Which of the following indicates that additional assistance is needed to transfer the patient from the bed to the stretcher? A. The patient is 5 feet 6 inches and weighs 120 lbs. B. The patient speaks and understands English. C. The patient received an injection of morphine 30 minutes ago for pain. D. You feel comfortable handling a patient of this size and level of cooperation.

C. The patient received an injection of morphine 30 minutes ago for pain.

Which of the following indicates that additional assistance is needed to transfer the patient from the bed to the stretcher? A. The patient is 5 feet 6 inches and weighs 120 lbs. B. The patient speaks and understands English. C. The patient received an injection of morphine 30 minutes ago for pain. D. You feel comfortable handling a patient of his size and with his level of cooperation.

C. The patient received an injection of morphine 30 minutes ago for pain. Correct

You are the charge nurse on a surgical unit. You are doing staff assignments for the 3-to-11 shift. Which patient do you assign to the licensed practical nurse (LPN)? A. The patient who transferred out of intensive care an hour ago B. The patient who requires teaching on new medications before discharge C. The patient who had a vaginal hysterectomy 2 days ago and is being discharged tomorrow D. The patient who is experiencing some bleeding problems following surgery earlier today

C. The patient who had a vaginal hysterectomy 2 days ago and is being discharged tomorrow Correct The patient with the vaginal hysterectomy is stable and requires care that is within the scope of the LPN. The other three patients need a higher level of care requiring assessment, support, and teaching that are the responsibilities of the registered nurse.

A nurse is sued for failure to monitor a patient appropriately after a procedure. Which of the following statements are correct about this lawsuit? (Select all that apply.) A. The nurse represents the plaintiff. B. The defendant must prove injury, damage, or loss. C. The person filing the lawsuit has the burden of proof. D. The plaintiff must prove that a breach in the prevailing standard of care caused an injury. .

C. The person filing the lawsuit has the burden of proof. Correct D. The plaintiff must prove that a breach in the prevailing standard of care caused an injury. Correct The patient as plaintiff must prove that the defendant nurse had a duty, breached the duty, and because of this breach caused the patient injury or damage. Awarded 0.0 points out of 2.0 possible points

he nurse is observing the student nurse perform a respiratory assessment on a patient. Which action by the student nurse requires the nurse to intervene? A. The student stands at a midline position behind the patient observing for position of the spine and scapula. B. The student palpates the thoracic muscles for masses, pulsations, or abnormal movements. C. The student places the bell of the stethoscope on the anterior chest wall to auscultate breath sounds. D. The student places the palm of the hand over the intercostal spaces and asks the patient to say "ninety-nine."

C. The student places the bell of the stethoscope on the anterior chest wall to auscultate breath sounds. Correct Breath sounds should be auscultated using the diaphragm of the stethoscope. Auscultate in a systematic pattern over the posterior and anterior chest wall.

During assessment of the skin, the nurse assesses a lesion on the arm of the patient. The lesion is irregularly shaped, elevated with edema, and about 3 cm. What type of lesion does the patient have? A. Nodule B. Macule C. Wheal D. Pustule

C. Wheal Correct A wheal is an irregularly shaped, elevated area or superficial localized edema. A wheal varies in size (e.g., hive, mosquito bite). A 1-cm firm, solid mass describes a nodule. A flat, brown area measuring 0.5 cm is a macule. A pus-filled circumscribed elevation of the skin is a pustule.

Nonvascular (without blood vessels) supporting connective tissue located chiefly in the joints and thorax, trachea, larynx, nose and ear

Cartilage

Is a set of guiding principles that all members of a profession accept.

Code of Ethics

Is part of the utilitarianism system of ethics that the value of something is determined by its usefulness. It's the outcome or consequence of action.

Consequentialism

When doing an assessment of a young woman who was in an automobile accident 6 months before, the nurse learns that the woman has vivid images of the crash whenever she hears a loud, sudden noise. The nurse recognizes this as ____________.

Correct Responses: "PTSD originates with a person's experiencing or witnessing a traumatic event and responding with intense fear or helplessness. The car accident is the traumatic ev, Posttraumatic stress disorder (PTSD), PTSD originates with a person's experiencing or witnessing a traumatic event and responding with intense fear or helplessness. The car accident is the traumatic ev, Posttraumatic stress disorder (PTSD)"

A patient with left-sided weakness asks his nurse, "Why are you walking on my left side? I can hold on to you better with my right hand." What would be your best therapeutic response? A. "Walking on your left side lets me use my right hand to hold on to your arm. In case you start to fall, I can still hold you." B. "Would you like me to walk on your right side so you feel more secure?" C. "Either side is appropriate, but I prefer the left side. If you like, I can have another nurse walk with you who will hold you on the right side." D. "By walking on your left side I can support you and help keep you from injury if you should start to fall. By holding your waist I would protect your shoulder if you should start to fall or faint.

D. "By walking on your left side I can support you and help keep you from injury if you should start to fall. By holding your waist I would protect your shoulder if you should start to fall or faint. Correct

A nurse is teaching a community group about ways to minimize the risk of developing osteoporosis. Which of the following statements made by a woman in the audience reflects a need for further education? A. "I usually go swimming with my family at the YMCA 3 times a week." B. "I need to ask my doctor if I should have a bone mineral density check this year." C. "If I don't drink milk at dinner, I'll eat broccoli or cabbage to get the calcium that I need in my diet." D. "I'll check the label of my multivitamin. If it has calcium, I can save money by not taking another pill.

D. "I'll check the label of my multivitamin. If it has calcium, I can save money by not taking another pill. "Correct Just because a multivitamin has calcium in it does not mean that the woman is receiving enough to meet her needs. She must know her requirement and make the decision based on that rather than on the value for calcium on the label.

Which statement made by the patient indicates an understanding about teaching related to early detection of colorectal cancer? A. "I'll make sure to schedule my colonoscopy annually after the age of 60." American Cancer Society guidelines state that for people of average risk, beginning at the age of 50, an annual fecal occult blood test is recommended. Flexible sigmoidoscopy and colonoscopy are recommended every 5 years in this population. A computed tomography (CT) colonoscopy is used every 5 years if recommended by the health care provider. B. "I'll make sure to have a computed tomography (CT) colonoscopy every 5 years." American Cancer Society guidelines state that for people of average risk, beginning at the age of 50, an annual fecal occult blood test is recommended. Flexible sigmoidoscopy and colonoscopy are recommended every 5 years in this population. A computed tomography (CT) colonoscopy is used every 5 years if recommended by the health care provider. C. "I'll make sure to have a flexible sigmoidoscopy every year once I turn 55." American Cancer Society guidelines state that for people of average risk, beginning at the age of 50, an annual fecal occult blood test is recommended. Flexible sigmoidoscopy and colonoscopy are recommended every 5 years in this population. A computed tomography (CT) colonoscopy is used every 5 years if recommended by the health care provider. D. "I'll make sure to have a fecal occult blood test annually once I turn 50."

D. "I'll make sure to have a fecal occult blood test annually once I turn 50." Correct American Cancer Society guidelines state that for people of average risk, beginning at the age of 50, an annual fecal occult blood test is recommended. Flexible sigmoidoscopy and colonoscopy are recommended every 5 years in this population. A computed tomography (CT) colonoscopy is used every 5 years if recommended by the health care provider.

The nurse plans to assess the patient's abstract reasoning. Which task should the nurse ask the patient to perform? A. "Tell me where you are." B. "What can you tell me about your illness?" C. "Repeat these numbers back to me: 7...5...8." D. "What does this mean: 'A stitch in time saves nine? '

D. "What does this mean: 'A stitch in time saves nine? ' " Correct Abstract reasoning requires cognitive functioning and the ability to identify relationships between concepts.

After a health care provider has informed a patient that he has colon cancer, the nurse enters the room to find the patient gazing out the window in thought. Which of the following are appropriate responses or actions of the nurse? (Select all that apply.) A. "I know another patient whose colon cancer was cured by surgery." B. Straighten the patient's bed and room C. "Have you thought about how you are going to tell your family?" D. "Would you like for me to sit down with you for a few minutes so you can talk about this?" E. Sit quietly with the patient

D. "Would you like for me to sit down with you for a few minutes so you can talk about this?"Correct E. Sit quietly with the patient Correct Sitting quietly or asking the patient if he would like you to sit down for a few minutes so he can talk are both effective. This provides the patient some quiet time, knowing that someone is there. Allowing the patient to talk allows the nurse to assess the patient's fears, knowledge, and perception of the situation, which is of utmost importance. The other responses are telling the patient what to do or giving reassurance, and the situation does not call for either of these.

You observe a nursing student taking a blood pressure (BP) on a patient. The patient's BP range over the past 24 hours is 132/64 to 126/72 mm Hg. The student used a BP cuff that was too narrow for the patient. Which of the following BP readings made by the student is most likely caused by the incorrect choice of BP cuff? A. 96/40 mm Hg B. 110/66 mm Hg C. 130/70 mm Hg D. 156/82 mm Hg

D. 156/82 mm Hg Correct When you use a blood pressure cuff that is too narrow or short, your patient will most likely have a BP reading that is higher than it really is; you will get a false-high reading.

The nurse is performing an abdominal assessment on a patient. In what order does the nurse perform the steps? 1. Percussion 2. Inspection 3. Auscultation 4. Palpation A. 4, 2, 3, 1 B. 1, 2, 3, 4 C. 3, 2, 4, 1 D. 2, 3, 4, 1

D. 2, 3, 4, 1 Correct The order of an abdominal examination differs slightly from previous assessments. Begin with inspection and follow with auscultation. By using auscultation before palpation there is less chance of altering the frequency and character of bowel sounds.

What are the correct steps to resolve an ethical dilemma on a clinical unit? Place the steps in correct order. 1. Clarify values. 2. Ask the question, Is this an ethical dilemma? 3. Verbalize the problem. 4. Gather information. 5. Identify course of action. 6. Evaluate the plan. 7. Negotiate a plan. A. 2, 4, 1, 5, 3, 7. 6 B. 2, 4, 3, 1, 5, 6, 7 C. 4, 1, 2, 3, 5, 7, 6 D. 2, 4, 1, 3, 5, 7, 6

D. 2, 4, 1, 3, 5, 7, 6 Correct In resolving an ethical dilemma, it first must be determined that an ethical dilemma exists. Then a systematic approach is needed to gather information, clarify values, verbalize the exact problem, identify a plan, negotiate elements of the plan, and evaluate the plan. ARCVINE

A woman who is a Jehovah's Witness has severe life-threatening injuries and is hemorrhaging following a car accident. The health care provider ordered 2 units of packed red blood cells to treat the woman's anemia. The woman's husband refuses to allow the nurse to give his wife the blood. What is the nurse's responsibility? A. Obtain a court order to give the blood B. Coerce the husband into giving the blood C. Call security and have the husband removed from the hospital D. Abide by the husband's wishes and inform the health care provider

D. Abide by the husband's wishes and inform the health care provider Correct Adult patients such as those who are Jehovah's Witnesses are able to refuse treatment for personal religious reasons

The registered nurse (RN) checks on a patient who was admitted to the hospital with pneumonia. The patient is coughing profusely and requires nasotracheal suctioning. Orders include an intravenous (IV) infusion of antibiotics. The patient is febrile and asks the RN if he can have a bath because he has been perspiring profusely. Which task is appropriate to delegate to the nursing assistant? A. Assessing vital signs B. Changing IV dressing C. Nasotracheal suctioning D. Administering a bed bath

D. Administering a bed bath Correct The bed bath is a skill and task within the knowledge level and tasks appropriate for a nursing assistant. The other tasks are the responsibility of the RN. Assessment, dressing change, and suctioning require assessment and skill that are within the scope of practice of the RN.

After having received 0.2 mg of naloxone (Narcan) intravenous push (IVP), a patient' s respiratory rate and depth are within normal limits. The nurse now plans to implement the following action: A. Discontinue all ordered opioids B. Close the room door to allow the patient to recover C. Administer the remaining naloxone over 4 minutes D. Assess patient's vital signs every 15 minutes for 2 hours

D. Assess patient's vital signs every 15 minutes for 2 hours Correct Reassess patients who receive naloxone every 15 minutes for 2 hours following drug administration because the duration of the opioid may be longer than the duration of the naloxone and respiratory depression may return.

A patient returning to the nursing unit after knee surgery is verbalizing pain at the surgical site. The nurse's first action is to: A. Call the patient's health care provider. B. Administer pain medication as ordered. C. Check the patient's vital signs. D. Assess the characteristics of the pain.

D. Assess the characteristics of the pain. Correct It is necessary to monitor pain on a regular basis along with other vital signs. It is important for the nurse to understand that pain assessment is not simply a number.

What is the best example of the nurse practicing patient advocacy? A. Seek out the nursing supervisor in conflicting procedural situations B. Document all clinical changes in the medical record in a timely manner C. Work to understand the law as it applies to an error in following standards of care D. Assess the patient's point of view and prepare to articulate it

D. Assess the patient's point of view and prepare to articulate it

What is the best example of the nurse practicing patient advocacy? A. Seek out the nursing supervisor in conflicting procedural situations B. Document all clinical changes in the medical record in a timely manner C. Work to understand the law as it applies to an error in following standards of care D. Assess the patient's point of view and prepare to articulate it

D. Assess the patient's point of view and prepare to articulate it Correct Seeking out the nursing supervisor, documenting clinical change in the medical record in a timely manner and working to understand the law as it applies to an error in following standards of care are not wrong; but advocacy generally refers to the nurse's ability to help speak for the patient.

A patient is admitted to a medical unit. The patient is fearful of hospitals. The nurse carefully assesses the patient to determine the exact fears and then establishes interventions designed to reduce these fears. In this setting how is the nurse practicing patient advocacy? A. Seeking out the nursing supervisor to talk with the patient B. Documenting patient fears in the medical record in a timely manner C. Working to change the hospital environment D. Assessing the patient's point of view and preparing to articulate it

D. Assessing the patient's point of view and preparing to articulate it Correct Assessing the patient's point of view and preparing to articulate it best reflects the concept of advocacy because it is standing up for the patient and having his or her views and wishes heard.

Which task is appropriate for a registered nurse (RN) to delegate to the nursing assistant? A. Explaining to the patient the preoperative preparation before the surgery in the morning B. Administering the ordered antibiotic to the patient before surgery C. Obtaining the patient's signature on the surgical informed consent D. Assisting the patient to the bathroom before leaving for the operating room

D. Assisting the patient to the bathroom before leaving for the operating room Correct Assisting the patient to the bathroom is a skill and task within the knowledge level and tasks appropriate for a nursing assistant. The other tasks are the responsibility of the RN. The RN is responsible for patient teaching, medication administration, and surgical consents.

The type of care management approach that coordinates and links health care services to patients and their families while streamlining costs and maintaining quality is: A. Primary nursing. B. Total patient care. C. Functional nursing. D. Case management.

D. Case management. Correct Case management is a care management approach that coordinates and links health care services to patients and their families while streamlining costs and maintaining quality. Case management is defined as "a collaborative process of assessment, planning, facilitation, and advocacy for options and services to meet an individual's health needs through communication and available resources to promote quality cost-effective outcomes."

A patient had a left-sided cerebrovascular accident 3 days ago and is receiving 5000 units of heparin subcutaneously every 12 hours to prevent thrombophlebitis. The patient is receiving enteral feedings through a small-bore nasogastric (NG) tube because of dysphagia. Which of the following symptoms requires the nurse to call the health care provider immediately? A. Pale yellow urine B. Unilateral neglect C. Slight movement noted on the R side D. Coffee ground-like aspirate from the feeding tube

D. Coffee ground-like aspirate from the feeding tube Correct When patients are receiving medications such as heparin or enoxaparin (Lovenox), you must assess for signs of bleeding. These include overt signs such as bleeding from their gums or covert signs, which can be detected by testing their stool or observing their aspirate from NG tubes for coffee ground-like matter. These are signs of bleeding in the gastrointestinal tract.

You are the night shift nurse caring for a newly admitted patient who appears to be confused. The family asks to see the patient's medical record. What is the priority nursing action? A. Give the family the record B. Discuss the issues that concern the family with them C. Call the nursing supervisor D. Determine from the medical record if the family has been granted permission by the patient to access his or her medical information

D. Determine from the medical record if the family has been granted permission by the patient to access his or her medical informationCorrect Family members do not have the right to private personal health information without the consent of the patient. Confidentiality protects private patient information once it has been disclosed in health care settings

A homeless man enters the emergency department seeking health care. The health care provider indicates that the patient needs to be transferred to the City Hospital for care. This action is most likely a violation of which of the following laws? A. Health Insurance Portability and Accountability Act (HIPAA) B. Americans with Disabilities Act (ADA) C. Patient Self-Determination Act (PSDA) D. Emergency Medical Treatment and Active Labor Act (EMTALA)

D. Emergency Medical Treatment and Active Labor Act (EMTALA) Correct The EMTALA requires that an emergency situation needs to be established and that the patient needs to be stabilized before a transfer is appropriate.

A homeless man enters the emergency department seeking health care. The health care provider indicates that the patient needs to be transferred to the City Hospital for care. This action is most likely a violation of which of the following laws? A. Health Insurance Portability and Accountability Act (HIPAA) B. Americans with Disabilities Act (ADA) C. Patient Self-Determination Act (PSDA) D. Emergency Medical Treatment and Active Labor Act (EMTALA) without triage completed

D. Emergency Medical Treatment and Active Labor Act (EMTALA) without triage completed Correct The EMTALA requires that an emergency situation needs to be established and that the patient needs to be stabilized before a transfer is appropriate.

The nurse summarizes the conversation with the patient to determine if the patient has understood him or her. This is what element of the communication process? A. Referent B. Channel C. Environment D. Feedback

D. Feedback

The patient for whom you are caring needs a liver transplant to survive. This patient has been out of work for several months and doesn't have health insurance or enough cash. What principles would be a priority in a discussion about ethics? A. Accountability because you as the nurse are accountable for the well-being of this patient B. Respect for autonomy because this patient's autonomy will be violated if he does not receive the liver transplant C. Ethics of care because the caring thing that a nurse could provide this patient is resources for a liver transplant D. Justice because the first and greatest question in this situation is how to determine the just distribution of resources

D. Justice because the first and greatest question in this situation is how to determine the just distribution of resources

1. The patient for whom you are caring needs a liver transplant to survive. This patient has been out of work for several months and doesn't have health insurance or enough cash. What principles would be a priority in a discussion about ethics? A. Accountability because you as the nurse are accountable for the well-being of this patient B. Respect for autonomy because this patient's autonomy will be violated if he does not receive the liver transplant C. Ethics of care because the caring thing that a nurse could provide this patient is resources for a liver transplant D. Justice because the first and greatest question in this situation is how to determine the just distribution of resources

D. Justice because the first and greatest question in this situation is how to determine the just distribution of resources Correct Accountability, respect or autonomy and ethics of care are not necessarily wrong, but they deflect attention from the less personal but more pertinent issue that is at stake in this situation: justice.

An older-adult patient has been bedridden for 2 weeks. Which of the following complaints by the patient indicates to the nurse that he or she is developing a complication of immobility? A. Loss of appetite B. Gum soreness C. Difficulty swallowing D. Left-ankle joint stiffness

D. Left-ankle joint stiffness Correct

A woman has severe life-threatening injuries and is hemorrhaging following a car accident. The health care provider ordered 2 units of packed red blood cells to treat the woman's anemia. The woman's husband refuses to allow the nurse to give his wife the blood for religious reasons. What is the nurse's responsibility? A. Obtain a court order to give the blood B. Coerce the husband into giving the blood C. Call security and have the husband removed from the hospital D. More information is needed about the wife's preference and if the husband has her medical power of attorney

D. More information is needed about the wife's preference and if the husband has her medical power of attorney Correct Adult patients such as those with specific religious objection are able to refuse treatment for personal religious reasons, but there need to be clear directions on who can make the decision.

Your patient is about to undergo a controversial orthopedic procedure. The procedure may cause periods of pain. Although nurses agree to do no harm, this procedure may be the patients only treatment choice. This example describes the ethical principle of A. Autonomy B. Fidelity C. Justice D. Nonmaleficence

D. Nonmaleficence

As the nurse, you need to complete all of the following. Which task do you complete first? A. Administer the oral pain medication to the patient who had surgery 3 days ago B. Make a referral to the home care nurse for a patient who is being discharged in 2 days C. Complete wound care for a patient with a wound drain that has an increased amount of drainage since last shift D. Notify the health care provider of the decreased level of consciousness in the patient who had surgery 2 days ago

D. Notify the health care provider of the decreased level of consciousness in the patient who had surgery 2 days ago Correct Decreased level of consciousness is a high priority. A high priority is an immediate threat to a patient's survival or safety such as a physiological episode of obstructed airway, loss of consciousness, or a psychological episode of an anxiety attack. Completing wound care would be the next priority, but it is not as critical as a change in consciousness. The other options are intermediate- or low-priority activities because they do not pose an immediate threat.

A staff nurse is talking with the nursing supervisor about the stress that she feels on the job. The supervising nurse recognizes that: A. Nurses who feel stress usually pass the stress along to their patients. B. A nurse who feels stress is ineffective as a nurse and should not be working. C. Nurses who talk about feeling stress are unprofessional and should calm down. D. Nurses frequently experience stress with the rapid changes in health care technology and organizational restructuring

D. Nurses frequently experience stress with the rapid changes in health care technology and organizational restructuring and when the situation seems out of their personal control.

A 52-year-old woman is admitted with dyspnea and discomfort in her left chest with deep breaths. She has smoked for 35 years and recently lost over 10 pounds. Her vital signs on admission are: HR 112, BP 138/82, RR 22, tympanic temperature 36.8°C (98.2°F), and oxygen saturation 94%. She is receiving oxygen at 2 L via a nasal cannula. Which vital sign reflects a positive outcome of the oxygen therapy? A. Temperature: 37°C (98.6°F) B. Radial pulse: 112 C. Respiratory rate: 24 D. Oxygen saturation: 96% E. Blood pressure: 134/78

D. Oxygen saturation: 96% Correct Oxygen therapy increases oxygen saturation. Temperature is not affected by the oxygen. There is no change in heart rate. Administering oxygen should decrease the respiratory rate. The decline in blood pressure is unlikely to be caused by oxygen.

The nurse is assessing a patient who returned 3 hours ago from a cardiac catheterization, during which the large catheter was inserted into the patient's femoral artery in the right groin. Which assessment finding would require immediate follow-up? A. Palpation of a femoral pulse with a heart rate of 76 B. Auscultation of a heart murmur over the left thorax C. Identification of mild bruising at the catheter insertion site D. Palpation of a right dorsalis pedis pulse with strength of +1

D. Palpation of a right dorsalis pedis pulse with strength of +1 Correct A weak pulse may indicate disruption of arterial flow and should be reported immediately. Mild bruising is normal, but if it increases in size, the femoral artery may be leaking, requiring further follow-up with the health care provider. Other findings are within normal limits and do not require notification. Awarded 0.0 points out of 1.0 possible points.

A patient returns to your postoperative unit following surgery for right shoulder rotator cuff repair. The licensed practical nurse (LPN) reports that she had difficulty obtaining the patient's heart rate from his right radial pulse. What is your best response? A. Assess the patient's apical pulse to obtain the heart rate. B. Obtain the heart rate from right and left radial sites. C. Obtain the heart rate using the oximeter probe. D. Perform a complete assessment of all pulses.

D. Perform a complete assessment of all pulses. Correct When an LPN reports that one pulse is difficult to obtain, first you need to assess the patient yourself and compare the quality of all pulses.

A patient with chronic low back pain who took an opioid around-the-clock (ATC) for the past year decided to abruptly stop the medication for fear of addiction. He is now experiencing shaking chills, abdominal cramps, and joint pain. The nurse recognizes that this patient is experiencing symptoms of: A. Addiction. B. Tolerance. C. Pseudoaddiction. D. Physical dependence.

D. Physical dependence. Correct Physical dependence is a state of adaptation that is manifested by a drug class specific withdrawal syndrome produced by abrupt cessation, rapid dose reduction, decreasing blood level of the drug, and/or administration of an antagonist.

The nurse puts elastic stockings on a patient following major abdominal surgery. The nurse teaches the patient that the stockings are used after a surgical procedure to: A. Prevent varicose veins. B. Prevent muscular atrophy. C. Ensure joint mobility and prevent contractures. D. Promote venous return to the heart.

D. Promote venous return to the heart. Correct Elastic stockings maintain external pressure on the lower extremities and assist in promoting venous return to the heart. This increase in venous return helps reduce the stasis of blood and in turn reduces the risk for deep vein thrombosis (DVT) formation in the lower extremities

A patient on week-long bed rest is now performing isometric exercises. Which nursing diagnosis best addresses the safety of this patient? A. Disturbed thought processes B. Impaired skin integrity C. Disturbed body image D. Risk for activity intolerance

D. Risk for activity intolerance

An older adult who was in a car accident and fractured his femur has been immobilized for 5 days. Which nursing diagnosis is related to patient safety when the nurse assists this patient out of bed for the first time? A. Chronic pain B. Impaired skin integrity C. Risk for ineffective cerebral tissue perfusion D. Risk for activity intolerance

D. Risk for activity intolerance Correct Patients on bed rest are at risk for activity intolerance, which increases patients' risk for falling.

The nurse plans care for a 16-year-old male, taking into consideration that stressors experienced most commonly by adolescents include which of the following? A. Loss of autonomy caused by health problems B. Physical appearance, family, friends, and school C. Self-esteem issues, changing family structure D. Search for identity with peer groups and separating from family

D. Search for identity with peer groups and separating from family are stressors most commonly experienced by adolescents. Loss of autonomy caused by health problems applies to the older adult. Physical appearance, family, friends and school apply to children. Self-esteem issues and a changing family structure apply to preadolescents.

When teaching a patient about transcutaneous electrical nerve stimulation (TENS), which information do you include? A. TENS works by causing distraction. B. TENS therapy does not require a health care provider's order. C. TENS requires an electrical source for use. D. TENS electrodes are applied near or directly on the site of pain.

D. TENS electrodes are applied near or directly on the site of pain. Correct TENS involves stimulation of the skin with a mild electrical current passed through external electrodes. The therapy requires a health care provider order. The TENS unit consists of a battery-powered transmitter, lead wires, and electrodes. Place the electrodes directly over or near the site of pain.

You are caring for Mr. Smith, who is facing amputation of his leg. During the orientation phase of the relationship, what would you do? A. Summarize what you have talked about in the previous sessions B. Review his medical record and talk to other nurses about how he is reacting C. Explore his feelings about losing his leg D. Talk with him about his favorite hobbies

D. Talk with him about his favorite hobbies

A patient newly diagnosed with type 2 diabetes says, "My blood sugar was just a little high. I don't have diabetes." The nurse responds: A. "Let's talk about something cheerful." B. "Do other members of your family have diabetes?" C. "I can tell that you feel stressed to learn that you have diabetes." D. With silence.

D. The nurse understands that denial is a defense mechanism that assists in coping with a shock. Therapeutic use of silence gives patients time to process their thoughts.

The statement that best explains the role of collaboration with others for the patient's plan of care is which of the following? A. The professional nurse consults the health care provider for direction in establishing goals for patients. B. The professional nurse depends on the latest literature to complete an excellent plan of care for patients. C. The professional nurse works independently to plan and deliver care and does not depend on other staff for assistance. D. The professional nurse works with colleagues and the patient's family to provide combined expertise in planning care.

D. The professional nurse works with colleagues and the patient's family to provide combined expertise in planning care.

The nurse encourages a patient with type 2 diabetes to engage in a regular exercise program primarily to improve the patient's: A. Gastric motility, thereby facilitating glucose digestion. B. Respiratory effort, thereby decreasing activity intolerance. C. Overall cardiac output, thereby resuming resting heart rate D. Use of glucose and fatty acids, thereby decreasing blood glucose level.

D. Use of glucose and fatty acids, thereby decreasing blood glucose level.

Which of the following nursing interventions should be implemented to maintain a patent airway in a patient on bed rest? A. Isometric exercises B. Administration of low-dose heparin C. Suctioning every 4 hours D. Use of incentive spirometer every 2 hours while awake

D. Use of incentive spirometer every 2 hours while awake Correct

You are transferring a patient who weighs 320 lb (145.5 kg) from his bed to a chair. The patient has an order for partial weight bearing as a result of bilateral reconstructive knee surgery. Which of the following is the best technique for transfer? A. Use a transfer board. B. Obtain a stand assist device. C. Implement a three-person carry. D. Use the ceiling-mounted lift.

D. Use the ceiling-mounted lift.

Defines actions as right or wrong based on "right making characteristics" such as fidelity to promises, truthfulness, and justice

Deontology

The study of conduct and character

Ethics

May even address issues beyond individual relationships such as ethical concerns about the structures within which individual caring occurs such as health care facilities.

Ethics of care

Refers to the agreement to keep promises

Fidelity

In healthcare promotes privacy and confidentiality (Widely respected in health care) Federal legislation known as the Health Insurance Portability and accountability act of 1996

HIPAA- health insurance portability and accountability act

Referring to Equal and Fair (Fairness)

Justice

The avoidance of harm or hurt

Nonmaleficence

Before transferring a patient from the bed to a stretcher, which assessment data do the nurse need to gather? (Select all that apply.) A. Patient's weight B. Patient's level of cooperation C. Patient's ability to assist D. Presence of medical equipment E. Nutritional intake

Patient's weight B. Patient's level of cooperation C. Patient's ability to assist D. Presence of medical equipment

The assessment of Mobility has 3 components

ROM - range of motion Gait exercise

Refers to a willingness to respect one's professional obligations and follow through on promises.

Responsibility As a nurse you are responsible for your actions and for the actions of those to whom you delegate tasks

Movement of bones and joints involves active processes that are carefully integrated to achieve coordination

Skeletal muscle

White glistening, fibrous bands of tissue that connect muscle to bone

Tendons

Value of something determined by its usefulness; main emphasis on the outcome

Utilitarianism

A personal belief about the worth of a given idea

Value

Personal belief about worth of given idea, attitude, custom, or object that sets standards that influence behavior

Value


Related study sets

Chapter 16 Accounting True/False Review

View Set

LSAT - LOGICAL REASONING - WRONG ANSWER TYPES

View Set

1.1 Assignment "Variables and Expressions"

View Set

Organizational Behavior Questions Review - Chapter 8

View Set